0% found this document useful (0 votes)
25 views

CAV Knowledge Check

This document provides information about lipids and fatty acids through multiple choice questions. It defines lipids, describes their functions, and classifies different types of lipids such as simple lipids, derived lipids, and fatty acids. The questions cover topics like lipid storage, essential fatty acids, fatty acid composition of different foods, and the roles of enzymes and hormones in lipid digestion.

Uploaded by

aucukagape
Copyright
© © All Rights Reserved
Available Formats
Download as PDF, TXT or read online on Scribd
0% found this document useful (0 votes)
25 views

CAV Knowledge Check

This document provides information about lipids and fatty acids through multiple choice questions. It defines lipids, describes their functions, and classifies different types of lipids such as simple lipids, derived lipids, and fatty acids. The questions cover topics like lipid storage, essential fatty acids, fatty acid composition of different foods, and the roles of enzymes and hormones in lipid digestion.

Uploaded by

aucukagape
Copyright
© © All Rights Reserved
Available Formats
Download as PDF, TXT or read online on Scribd
You are on page 1/ 109

OLM #1 - MCB

By definition, lipids are all the following except:


● A large group of organic substances
● Highly variable in structure
● Highly saturated forms of energy
● Insoluble in water
● Insoluble in organic solvents
Correct Answer: Insoluble in organic solvents
Lipids function in all the following except:
● Act as second messengers
● Act as thermal insulators
● Facilitate storage of water-soluble vitamins
● Increase solubility of other lipids
● Facilitate nerve conduction
● Synthesis of steroids
Correct Answer: Facilitate storage of water-soluble vitamins
Lipids classified as simple lipids include:
● Fats, waxes, oils, and phospholipids
● Phospholipids, glycolipids, and fats
● Glycolipids, fats, and waxes
● Waxes, oils, and fats
● Glycolipids, phospholipids, fats, and waxes
Correct Answer: Waxes, oils, and fats
Derived lipids include all the following except:
● Cholesterol
● Bile acids
● Triacyl glycerides
● Adrenocortical hormones
● Cholesterol esters
● Eicosanoids
Correct Answer: Triacyl glycerides
Based upon the included diagram, which of the designated carbons (A-E) is referred to as the
Omega carbon?

Correct Answer: C
Which of the following fatty acids is a four-carbon fatty acid which can be synthesized in the
human body?
● Valeric acid
● Acetic acid
● Propionic acid
● Butyric acid
● Palmitic acid
Correct Answer: Butyric acid
Which of the following fatty acids is a five-carbon fatty acid produced under anerobic conditions
by the intestinal bacteria?
● Valeric acid
● Acetic acid
● Propionic acid
● Butyric acid
● Palmitic acid
Correct Answer: Valeric acid
Medium-chain fatty acids are generally considered to be…
● Fewer than six carbons in length
● Between six and fourteen carbons in length
● Between fourteen and twenty carbons in length
● Longer than twenty carbons in length
● Derived from twenty carbon fatty acids
Correct Answer: Between six and fourteen carbons in length
Which of the following fatty acids is an essential ω-3 fatty acid?
● Linoleic acid
● γ-linolenic acid
● Arachidonic acid
● α-Linolenic acid
● Eicosapentaenoic acid
Correct Answer: α-Linolenic acid
While not technically an “essential fatty acid”, when the ω-6 fatty acid linoleic acid becomes
limited in the body, _____ becomes essential and must be taken up as a dietary lipid because it
can only be synthesized from linoleic acid.
● Docosahexanoic acid
● γ-linolenic acid
● Arachidonic acid
● α-Linolenic acid
● Eicosapentaenoic acid
Correct Answer: Arachidonic acid
Some fatty acids which are contained in certain seeds and nuts are considered to be “cyclic fatty
acid” because they have a five-carbon ring-structure on the _______ end of the molecule. The
fatty acid, __________, is one of this type of lipid.
● Omega end, Phytanic acid
● Delta end, Pristanic acid
● Omega end, Chaulmoogric acid
● Delta end, Chaulmoogric acid
● Omega end, Pristanic acid
● Delta end, Pristanic acid
Correct Answer: Omega end, Chaulmoogric acid
Which of the following is considered to be an ω-9 fatty acid
Correct Answer: B
One of the reasons that fats are a much-preferred chemical form for energy storage is because
______.
● Each gram of stored glycogen requires hydration with approximately 5 grams of H2O
for storage.
● Fats much be hydrated with approximately 2 grams of H2O for storage.
● While oxidation of carbohydrates releases over twice the amount of energy released
from fatty acid oxidation, fatty acids do not require hydration.
● Fatty acids can be stored in the anhydrous form and do not require hydration.
● Metabolism of the amino acids in a gram of protein releases over five times less energy
than the oxidation of a gram of fatty acids.
Correct Answer: Fatty acids can be stored in the anhydrous form and do not require hydration.
Assuming the typical human requires 2000 kcal of energy per day, and only considering caloric
requirements, a person consuming only pure olive oil (oleic acid) would need to consume
approximately __________ of olive oil each day to meet their minimal energetic requirements.
● 25 grams
● 100 grams
● 175 grams
● 225 grams
● 450 grams
Correct Answer: 225 grams
Different tissues and organs access energy from sources based upon their metabolic needs and
capabilities. Complete the table provided below appropriately based upon the energy utilization
for the Liver, Neurons, Heart, Muscle and Erythrocytes.
Organs Glucose Fatty Acids Ketone Bodies

Liver Primary Secondary Cannot use

Neurons Primary Cannot use Secondary (starvation)

Heart Secondary Primary Minimal


Muscle Fast twitch Slow twitch Secondary

RBCs Only source Cannot use Cannot use

In the fasting state, Triacylglycerol is broken down by hormone sensitive lipase and other lipases
to free fatty acids and glycerol. Under fed conditions, the hormone sensitive lipase is inactive.
Activation is regulated by intracellular cAMP concentrations. When intracellular cAMP
concentrations increase _____________ is activated, which in turn activates the hormone
sensitive lipase by ______________.
● Lipase Phosphatase I (LPI), dephosphorylation
● Lipase Phosphatase I (LPI), phosphorylation
● Protein Kinase A (PKA), dephosphorylation
● Protein Kinase A (PKA), phosphorylation
● Phosphoprotein Phosphatase (PPP), dephosphorylation
● Phosphoprotein Phosphatase (PPP), phosphorylation
Correct Answer: Protein Kinase A (PKA), phosphorylation
Diet composition varies from region to region in the world. In the western world, an average
adult consumes between 60 and 150g of fat per day. Of this _________ is typically composed of
triglycerides.
● 50%
● 65%
● 75%
● 85%
● >90%
Correct Answer: >90%
Lingual and gastric lipases are acid-stable enzymes capable of breaking down some TAGs. Up to
one third of the dietary fat is said to be hydrolyzed in the stomach by lingual lipases. Lingual
lipases are capable of predominantly releasing which of the following metabolites from the
TAGs they digest?
● Free short and medium chain fatty acids and 1-monoacylglycerols (1-MAGs)
● Free short and medium chain fatty acids and 2-monoacylglycerols (2-MAGs)
● Free short and medium chain fatty acids and 3-monoacylglycerols (3-MAGs)
● Free short and medium chain fatty acids and 1,2-diacylglycerols (1,2-DAGs)
● Free short and medium chain fatty acids and 2,3-diacylglycerols (2,3-DAGs)
Correct Answer: Free short and medium chain fatty acids and 1,2-diacylglycerols (1,2-DAGs)
Free Short- (SCFA) and Medium Chain Fatty Acids (MCFAs) are important dietary components
of the neonate diet because _________.
● They are a major component of breast milk, can be liberated by hormone sensitive
lipases and they are easily packed in chylomicrons for transport to the liver for further
digestion.
● They are a minor component of breast milk, are liberated by hormone sensitive lipases
and they are easily packed in chylomicrons for transport to the liver for further
digestion.
● They are a major component of breast milk, can be liberated by lingual lipases and they
can diffuse directly into the blood stream for transport to the liver for further digestion.
● They are a major component of breast milk, can be liberated by lingual lipases and they
are easily packed in chylomicrons for transport to the liver for further digestion.
Correct Answer: They are a major component of breast milk, can be liberated by lingual lipases
and they can diffuse directly into the blood stream for transport to the liver for further digestion.
Endogenous lipids are most easily broken down by the following lipases.
● Lingual-, Gastric-, Hormone sensitive- and Pancreatic lipases
● Lipoprotein-, Gastric-, Hormone sensitive- and Pancreatic lipases
● Hepatic-, Gastric-, Lingual- and Lipoprotein lipases
● Hepatic-, Lipoprotein- and Hormone-sensitive lipases
● Hepatic-, Lipoprotein-, Gastric- and Hormone-sensitive lipases
Correct Answer: Hepatic-, Lipoprotein- and Hormone-sensitive lipases
Following ingestion of a typical meal, the peptide hormone cholecystokinin is released into the
lumen of the gut to further facilitate digestion. All of the following are true except:
● Cholecystokinin is produced by the gut endocrine cells
● Cholecystokinin release is stimulated by partially digested proteins and lipids
● Cholecystokinin causes gallbladder contractions resulting in the secretion of bile
● Cholecystokinin release reduces gastric mobility in the stomach
● Cholecystokinin is released by the mucosal cells of the stomach
● Cholecystokinin release stimulates the exocrine pancreas to secrete digestive enzymes
Correct Answer: Cholecystokinin is released by the mucosal cells of the stomach.
Bile acids are largely produced in the liver through cholesterol metabolism. Primary and
secondary bile acids are distinct. Which of the following are considered primary bile acids?
(Select all that apply.)
● Cholic acid
● Deoxycholic acid
● Chenodeoxycholic acid
● Lithocholic acid
● Lanosterol
Correct Answers: Cholic acid, Chenodeoxycholic acid
Bile acids are largely produced in the liver through cholesterol metabolism. Primary and
secondary bile acids are distinct. Secondary bile acids are metabolic products of the gut
microflora. They are partially dehydroxylated metabolites of the primary bile acids. Which of the
following are considered secondary bile acids? (Select all that apply.)
● Cholic acid
● Deoxycholic acid
● Chenodeoxycholic acid
● Lithocholic acid
● Lanosterol
Correct Answers: Deoxycholic acid, Lithocholic acid
Once the lipid-rich meal reaches the small intestine, pancreatic lipase begins to play a major role
in lipid digestion. Colipase functions as a protein coenzyme in a 1:1 ratio for optimal pancreatic
lipase activity. All of the following are true except:
● Colipase serves to anchor the pancreatic lipase to the surface of the lipid-rich micelle
● Colipase is secreted from the pancreas as pro-colipase and then activated by trypsin
cleavage to colipase
● Complete lipid digestion by pancreatic lipase releases one molecule of free fatty acid
and one molecule of 1,2-diacylglycerol
● Complete lipid digestion by pancreatic lipase releases two molecules of free fatty acid
and one molecule of 2-monoacylglycerol
● Initial lipid digestion by pancreatic lipase releases two molecule of free fatty acid and
one molecule of 1,2 diacylglycerol
Correct Answer: Complete lipid digestion by pancreatic lipase releases two molecules of free
fatty acid and one molecule of 2-monoacylglycerol.
Phospholipase A2 is an important enzyme involved in lipid digestion. Which is an accurate
description of the enzymatic activity of Phospholipase A2?
● Phospholipase A2 cleaves triacylglycerols to release free fatty acid and glycerol
● Phospholipase A2 cleaves 1,2 diacylglycerol to release free fatty acid and glycerol
● Phospholipase A2 cleaves 2-monoacylglycerol to release free fatty acid and glycerol
● Phospholipase A2 cleaves phospholipids to release free fatty acid and glycerol
● Phospholipase A2 cleaves phospholipids to release free fatty acid and lysophospholipid
Correct Answer: Phospholipase A2 cleaves phospholipids to release free fatty acid and
lysophospholipid.
Within the small intestine, after lipid digestion of the lipids contained within micelles, the
“mixed micelles” produced are nearly 100x smaller than the micelles which result from the
initial emulsification process. The contents of mixed micelles predominantly include:
● Monoacylglycerols, phospholipids, free fatty acids, cholesterol, fat soluble vitamins and
bile acids
● Monoacylglycerols, lysophospholipids, free fatty acids, cholesterol, fat soluble vitamins
and bile acids
● Triacylglycerols, lysophospholipids, free fatty acids, cholesterol, fat soluble vitamins
and bile acids
● Monoacylglycerols, lysophospholipids, free fatty acids, cholesterol esters, fat soluble
vitamins and bile acids
● Monoacylglycerols, lysophospholipids, free fatty acids, cholesterol, esters of fat-soluble
vitamins and bile acids
Correct Answer: Monoacylglycerols, lysophospholipids, free fatty acids, cholesterol, fat soluble
vitamins and bile acids
The lipid content of mixed micelles can be absorbed freely through the plasma membrane of
small intestine epithelial cells. Which of the lipids listed below is not a major lipid component of
the mixed micelle lipid contents which readily enter the epithelial cells?
● Cholesterol
● Phospholipids
● Monoacylglycerols
● Lysophospholipids
● Free fatty acids
Correct Answer: Phospholipids
Following entry into the epithelial cells of the small intestines, the lipids are converted back to
their undigested forms and packaged in much larger structures referred to as chylomicrons. Final
formation of the chylomicrons occurs in the:
● Cytoplasm of the epithelial cells
● Smooth ER of the epithelial cells
● Rough ER of the epithelial cells
● Golgi of the epithelial cells
● Mitochondria of the epithelial cells
Correct Answer: Golgi of the epithelial cells
Chylomicrons are assembled in the intestinal mucosa from reassembled lipid components of the
ingested lipid-rich meal. Chylomicrons are also sometimes referred to as ultra-low-density
lipoproteins (ULDL). Other than the reassembled lipids, chylomicrons also contain
approximately 1-2% protein. A protein specific to chylomicrons and not found in other
lipoproteins is:
● ApoA1
● ApoB48
● ApoB100
● ApoC
● ApoE
Correct Answer: ApoB48
The enterohepatic circulation pathway is important for carrying the vast majority of _________
which are reabsorbed in the ileum and returned back to the liver.
● Undigested triacylglycerols
● Partially digested triacylglycerols
● Cholesterol
● 1,2 diacylglycerols
● Bile salts
● Lipoproteins
Correct Answer: Bile salts
Patients administered Orlistat on a long-term basis may have a common symptom which is also
seen in some cases of Crohn’s or Celiac disease, steatorrhea. This is because:
● Orlistat directly reduces lipase secretion
● Orlistat directly inhibits lipase activity
● Orlistat directly reduces release of intraduodenal bile acids
● Orlistat directly inhibits the action of intraduodenal bile acids
● Orlistat directly inhibits mucosal uptake and re-esterification of lipids
Correct Answer: Orlistat directly inhibits lipase activity
OLM #2 - Anatomy
The second costal cartilage can be located by palpating the:
● Costal margin
● Sternal angle
● Sternal notch
● Sternoclavicular joint
● Xiphoid process
Correct answer: Sternal angle
The tubercle of the 7th rib articulates with which structure?
● Body of vertebra T6
● Body of vertebra T7
● Body of vertebra T8
● Transverse process of vertebra T6
● Transverse process of vertebra T7
Correct answer: Transverse process of vertebra T7
While observing in the OR, you see the resident insert a needle through the body wall just above
the ninth rib in the mid-axillary line. She was trying to enter the:
● Costodiaphragmatic recess
● Costomediastinal recess
● Cupola
● Hilar reflection
● Pulmonary ligament
Correct answer: Costodiaphragmatic recess
A 37-year-old man was pinned against the wall of a loading dock when a forklift backed into
him, breaking two ribs and causing a right-sided hemothorax. A negative pressure drain (chest
tube) must be inserted into the pleural space in order to remove blood from his pleural cavity.
What would be a typical location for placement of the chest tube?
● Right fourth intercostal space in the midclavicular line
● The left intercostal space adjacent to the xiphoid process
● Right sixth intercostal space in the anterior axillary line
● Left seventh intercostal space in the middle axillary line
● Right 8th intercostal space in the subscapular line
Correct answer: Right 8th intercostal space in the subscapular line
Which of the following layers provides a natural cleavage plane for surgical separation of the
costal pleura from the thoracic wall?
● Deep fascia
● Endothoracic fascia
● Parietal pleura
● Visceral pleura
● Transversus thoracis muscle fascia
Correct answer: Endothoracic fascia
All layers of the thoracic wall (skin, intercostal muscles, endothoracic fascia) are innervated by
which of the following nerves?
● Dorsal primary rami
● Intercostal nerves
● Lateral pectoral nerves
● Medial pectoral nerves
● Thoracodorsal nerves
Correct answer: Intercostal nerves
In the drawing below, the 5 openings of the thoracic diaphragm lettered A, B, C, D, & E. Which
one transmits the azygos vein?
Correct answer: D
Which of the following structures accompany the esophagus through the diaphragm?
● The phrenic nerves
● The splanchnic nerves
● The sympathetic trunks
● The vagus nerves
Correct answer: The vagus nerves
A 49-year-old woman is diagnosed with having a large lump in her right breast. Lymph from the
cancerous breast drains first into which of the following lymph node groups?
● Humeral
● Anterior (pectoral)
● Supraclavicular
● Apical
● Posterior (subscapular)
Correct answer: Anterior (pectoral)
A premenopausal 51-year-old female describes the recent discovery of a lump in her right breast.
Physical examination reveals a mass of approximately 2.5 cm in the upper lateral breast
quadrant. There is some dimpling of the skin overlying the mass. This dimpling is due to:
● Traction on connective tissue septae by the expanding mass
● Influence of cyclical hormonal changes
● Displacement of glandular tissue by the mass
● Increased vasculature to the mass
● Decreased fat in the area of the mass
Correct answer: Traction on connective tissue septae by the expanding mass
While observing a mastectomy on a 60-year-old female patient, a medical student was asked by
the surgeon to help tie off the arteries that supply the medial side of the breast. The artery that
gives origin to these small branches is the:
● Internal thoracic
● Lateral thoracic
● Posterior intercostal
● Superior epigastric
● Thoracoacromial
Correct answer: Internal thoracic
OLM #3 - MCB
The initial steps in the synthesis of ketone bodies is the conversion of stored triglycerides to free
fatty acids by:
● Lingual lipase
● Gastric lipase
● Hormone sensitive lipase
● Pancreatic lipase
● Skeletal muscle lipase
Correct Answer: Hormone sensitive lipase
Omega oxidation of fatty acids occurs in the:
● Mitochondria
● Peroxisomes
● Rough ER
● Smooth ER
● Lysosomes
Correct Answer: Smooth ER
Beta oxidation of very long chain fatty acids (VLCFA) occurs in the:
● Mitochondria
● Lysosomes
● Rough ER
● Smooth ER
● Peroxisomes
Correct Answer: Peroxisomes
Beta oxidation of medium and long chain fatty acids occurs in the:
● Mitochondria
● Peroxisomes
● Rough ER
● Smooth ER
● Lysosomes
Correct Answer: Mitochondria
Alpha oxidation of branched chain fatty acids occurs in the:
● Mitochondria
● Peroxisomes
● Rough ER
● Smooth ER
● Lysosomes
Correct Answer: Peroxisomes
The enzyme required for movement of free long chain fatty acyl-CoA into the mitochondrial
inter-membrane space is:
● Fatty Acyl-CoA Synthetase
● Carnitine Acyltransferase 1 (CAT-1/CPT-1)
● Carnitine Translocase
● Carnitine Acyltransferase II (CAT-II/CPT-II)
● Medium Chain Acyl Dehydrogenase
Correct Answer: Fatty Acyl-CoA Synthetase
The enzyme which converts free long chain fatty acids into fatty acylcarnitine in the
mitochondrial inter-membrane space is:
● Fatty Acyl-CoA Synthetase
● Carnitine Acyltransferase 1 (CAT-1/CPT-1)
● Carnitine Translocase
● Carnitine Acyltransferase II (CAT-II/CPT-II)
● Medium Chain Acyl Dehydrogenase
Correct Answer: Carnitine Acyltransferase 1 (CAT-1/CPT-1)
The enzyme which moves fatty acylcarnitine across the mitochondrial inter-membrane into the
matrix is:
● Fatty Acyl-CoA Synthetase
● Carnitine Acyltransferase 1 (CAT-1/CPT-1)
● Carnitine Translocase
● Carnitine Acyltransferase II (CAT-II/CPT-II)
● Medium Chain Acyl Dehydrogenase
Correct Answer: Carnitine Translocase
The enzyme which converts fatty acylcarnitine in the mitochondrial matrix to Fatty acyl-CoA is:
● Fatty Acyl-CoA Synthetase
● Carnitine Acyltransferase 1 (CAT-1/CPT-1)
● Carnitine Translocase
● Carnitine Acyltransferase II (CAT-II/CPT-II)
● Medium Chain Acyl Dehydrogenase
Correct Answer: Carnitine Acyltransferase II (CAT-II/CPT-II)
Carnitine deficiency can be categorized as Primary or Secondary. Which of the following is not a
direct cause of secondary carnitine deficiency?
● Liver failure
● Kidney failure
● Malnutrition
● Strict vegetarian diet
● Dialysis treatment
● Defective plasma membrane transporter
Correct Answer: Defective plasma membrane transporter
Which of the following metabolic myopathies affects the slow twitch muscle fibers, results in
increased lactate production, accumulation of neutral lipids in the muscle and Acyl carnitines in
the urine?
● McArdle disease
● Hunter syndrome
● Krabbe disease
● CAT-II deficiency
● Wilson’s disease
Correct Answer: CAT-II deficiency
β-oxidation of fatty acyl-CoA in the matrix of the mitochondria requires a repeated four-reaction
process. The first step of four requires _____ and is catalyzed by the enzyme _____.
● FAD, acyl-CoA dehydrogenase
● FADH2, acyl-CoA dehydrogenase
● NAD+, 3-hydroxy acyl-CoA dehydrogenase
● NADH, 3-hydroxy acyl-CoA dehydrogenase
● FAD, Enoyl-CoA dehydrogenase
● H2O, Enoyl-CoA dehydrogenase
● NAD+, acyl-CoA acyltransferase/b-keto Thiolase
● Coenzyme A, acyl-CoA acyltransferase/b-keto Thiolase
● Acetyl-CoA, acyl-CoA acyltransferase/b-keto Thiolase
Correct Answer: FAD, acyl-CoA dehydrogenase
β-oxidation of fatty acyl-CoA in the matrix of the mitochondria requires a repeated four-reaction
process. The second step of four requires _____ and is catalyzed by the enzyme _____.
● FAD, acyl-CoA dehydrogenase
● FADH2, acyl-CoA dehydrogenase
● NAD+, 3-hydroxy acyl-CoA dehydrogenase
● NADH, 3-hydroxy acyl-CoA dehydrogenase
● FAD, Enoyl-CoA hydratase
● H2O, Enoyl-CoA hydratase
● NAD+, acyl-CoA acyltransferase/β-keto Thiolase
● Coenzyme A, acyl-CoA acyltransferase/β-keto Thiolase
● Acetyl-CoA, acyl-CoA acyltransferase/β-keto Thiolase
Correct Answer: H 2O, Enoyl-CoA hydratase
β-oxidation of fatty acyl-CoA in the matrix of the mitochondria requires a repeated four-reaction
process. The third step of four requires _____ and is catalyzed by the enzyme _____.
● FAD, acyl-CoA dehydrogenase
● FADH2, acyl-CoA dehydrogenase
● NAD+, 3-hydroxy acyl-CoA dehydrogenase
● NADH, 3-hydroxy acyl-CoA dehydrogenase
● FAD, Enoyl-CoA hydratase
● H2O, Enoyl-CoA hydratase
● NAD+, acyl-CoA acyltransferase/β-keto Thiolase
● Coenzyme A, acyl-CoA acyltransferase/β-keto Thiolase
● Acetyl-CoA, acyl-CoA acyltransferase/β-keto Thiolase
Correct Answer: NAD +, 3-hydroxy acyl-CoA dehydrogenase
β-oxidation of fatty acyl-CoA in the matrix of the mitochondria requires a repeated four-reaction
process. The fourth step of four requires _____ and is catalyzed by the enzyme _____.
● FAD, acyl-CoA dehydrogenase
● FADH2, acyl-CoA dehydrogenase
● NAD+, 3-hydroxy acyl-CoA dehydrogenase
● NADH, 3-hydroxy acyl-CoA dehydrogenase
● FAD, Enoyl-CoA hydratase
● H2O, Enoyl-CoA hydratase
● NAD+, acyl-CoA acyltransferase/β-keto Thiolase
● Coenzyme A, acyl-CoA acyltransferase/β-keto Thiolase
● Acetyl-CoA, acyl-CoA acyltransferase/β-keto Thiolase
Correct Answer: Coenzyme A, acyl-CoA acyltransferase/β-keto Thiolase
If the energy of FA activation is ignored, the complete oxidation of palmitic acid via b-oxidation
yields _____. If these all feed into the TCA cycle and electron transport chain, ____ ATP can be
produced.
● 6 FADH2, 6 NADH, and 7 Acetyl-CoA. 106
● 6 FADH2, 6 NADH, and 7 Acetyl-CoA. 108
● 7 FADH2, 7 NADH, and 8 Acetyl-CoA. 106
● 7 FADH2, 7 NADH, and 8 Acetyl-CoA. 108
● 8 FADH2, 8 NADH, and 9 Acetyl-CoA. 108
Correct Answer: 7 FADH 2, 7 NADH, and 8 Acetyl-CoA. 108
The oxidation of unsaturated fatty acids cannot proceed via mitochondrial β-oxidation until the
double bonds have been removed by the enzyme(s):
● Desaturase and Dismutase
● Desaturase and Isomerase
● Isomerase and Dismutase
● Isomerase and reductase
● Desaturase and reductase
Correct Answer: Isomerase and reductase
The oxidation of odd-chain fatty acids proceeds via mitochondrial β-oxidation until the
three-carbon molecule _______ is produced. Then through a series of two reactions, that product
is converted to __________ which can feed into the citric acid cycle.
● Propionate, Methyl-malonyl-CoA
● Propionate, Propionate
● Propionate, Succinyl-CoA
● Propionyl-CoA, Succinyl-CoA
● Propionyl-CoA, Methyl-malonyl-CoA
Correct Answer: Propionyl-CoA, Succinyl-CoA
Because very long chain fatty acids cannot enter the mitochondria, they must first be oxidized in
the _____, to form LCFA and MCFA through the process of _______.
● SER, alpha oxidation
● SER, omega oxidation
● Peroxisomes, beta oxidation
● Peroxisomes, alpha oxidation
● RER, alpha oxidation
Correct Answer: Peroxisomes, beta oxidation
Because branched chain fatty acids cannot be oxidized through beta oxidation in the
mitochondria, they must first be oxidized in the _____, to form LCFA and MCFA through the
process of _______.
● SER, alpha oxidation
● SER, omega oxidation
● Peroxisomes, beta oxidation
● Peroxisomes, alpha oxidation
● RER, alpha oxidation
Correct Answer: Peroxisomes, alpha oxidation
When beta oxidation in the mitochondria is insufficient, free fatty acids can accumulate. When
this occurs, ______ which occurs in the ______ is used to break down the excess free fatty acids.
● alpha oxidation, SER
● omega oxidation, SER
● beta oxidation, Peroxisomes
● alpha oxidation, Peroxisomes
● alpha oxidation, RER
Correct Answer: omega oxidation, SER
Medium Chain Acy-CoA Dehydrogenase Deficiency (MCAD) is a very serious disease. It is the
leading cause of SIDS and inborn error in FA oxidation. Which of the following is not a
symptom on MCAD?
● Decreased beta oxidation
● Hyperglycemia
● Metabolic acidosis
● Acute abdominal pain
● Aspiration in the crib
Correct Answer: Hyperglycemia
Zellweger Syndrome is an autosomal recessive disease spectrum which impacts the brain,
kidneys, and liver. This disease effects the generation of the ________ and results in the
accumulation of ______.
● SER, VLFAs and Branched Chain FAs
● RER, VLFAs and Branched Chain FAs
● Peroxisomes, VLFAs and Branched Chain FAs
● Golgi, VLFAs and Branched Chain FAs
● Mitochondria, VLFAs and Branched Chain FAs
Correct Answer: Peroxisomes, VLFAs and Branched Chain FAs
X-linked Adrenoleukodystrophy is a recessive disease which largely impacts the brain, adrenal
cortex and Leydig cells of the testis. This disease results in the accumulation of VLCFAs because
of a defective _______ protein affects the import of VLCFAs into the ______.
● membrane transporter, mitochondria
● membrane transporter, peroxisome
● membrane transporter, SER
● membrane transporter, RER
● membrane transporter, lysosome
Correct Answer: membrane transporter, peroxisome
Symptoms of Adult Refsum disease can include progressive muscle weakness, peripheral
polyneuropathy, cerebellar ataxia, and ichthyosis. Retinitis pigmentosa and anosmia (loss of
smell) are also common. Adult Refsum disease results from a deficiency in phytanoyl-CoA
hydroxylase which is required for ________ and results in the accumulation of _______ fatty
acids.
● Alpha oxidation, VLC
● Alpha oxidation, branch chain
● Alpha oxidation, saturated
● Peroxisomal beta oxidation, VLC
● Peroxisomal beta oxidation, branch chain
● Peroxisomal beta oxidation, saturated
Correct Answer: Alpha oxidation, branch chain
Ketone body synthesis occurs in the _____. The levels of _____ in this organ is the sole driver of
ketogenesis.
● Spleen, glycolysis
● Liver, glycolysis
● Kidneys, glycolysis
● Spleen, beta oxidation
● Liver, beta oxidation
● Kidneys, beta oxidation
Correct Answer: Liver, beta oxidation
The flooding of fats mobilized from the adipose tissue during starvation leads to increased
production of acetyl-CoA. This results in ______ pyruvate dehydrogenase activity, _____
pyruvate carboxylase activity and ______ gluconeogenesis.
● Increased, increased, increased
● Increased, decreased, increased
● Decreased, increased, increased
● Decreased, increased, decreased
● Increased, decreased, decreased
Correct Answer: Decreased, increased, increased
During fasting and starvation, β-oxidation produces NADH and ATP which results in inhibition
of the citric acid cycle at isocitrate-dehydrogenase. This results in the accumulation of _____,
ultimately resulting in the synthesis of _______.
● FFA, Acetyl-CoA
● FFA, Pyruvate
● Pyruvate, Ketone bodies
● Acetyl-CoA, Ketone bodies
● Pyruvate, Ketone bodies
Correct Answer: Acetyl-CoA, Ketone bodies
Except for ______, all of the following are considered to be ketone bodies.
● Acetone
● Acetic acid
● Acetoacetic acid
● Beta-hydroxybutyric acid
● 3-hydroxbutyric acid
● Beta-hydroxybutyrate
Correct Answer: Acetic acid
The final fate of ketone bodies which are not metabolized end up deposited in the ______ and
______.
● Liver and blood
● Liver and extrahepatic tissues
● Expired air and urine
● Extrahepatic tissues and acetyl-CoA
● Liver and acetyl-CoA
Correct Answer: Expired air and urine
The rate limiting step in ketone body synthesis involves the enzyme, _____ which combines an
acetyl-CoA with Acetoacetyl-CoA to produce ________.
● HMG-CoA synthase, HMG-CoA
● Acetyl-CoA acetyl transferase, acetyl-CoA
● HMG-CoA lyase, Acetoacetate
● 3-hydroxybutyrate dehydrogenase, acetyl-CoA
Correct Answer: HMG-CoA synthase, HMG-CoA
During the fed-state, the two metabolizable ketone bodies circulate in the blood plasma at an
approximate ratio of ____. Their typical plasma concentration in this state is _____.
● 1:1, <0.1mmol/L
● 1:1, 1mmol/L
● 1:1, 5-10mmol/L
● 4:1, <0.1mmol/L
● 4:1, 1mmol/L
● 4:1, 5-10mmol/L
Correct Answer: 4:1, <0.1mmol/L
In the diabetic ketoacidosis state, the two metabolizable ketone bodies circulate in the blood
plasma at an approximate ratio of ____. Their typical plasma concentration in this state is _____.
● 4:1, <0.1mmol/L
● 4:1, 1mmol/L
● 4:1, 5-10mmol/L
● 10:1, <0.1mmol/L
● 10:1, 1mmol/L
● 10:1, 5-10mmol/L
Correct Answer: 10:1, 5-10mmol/L
Ketogenesis is somewhat under hormonal control. During ketosis, plasma ketone body
concentrations can rise leading to ketonemia. Excess ketone bodies can be eliminated in the urine
leading to ketouria. ____ inhibit(s) ketogenesis and ketosis. ____ stimulate(s) ketogenesis and
ketosis.
● Acetyl-CoA, Free fatty acids
● Acetyl-CoA, glucose
● Glucagon, Insulin
● Insulin, Glucagon
● Acetoacetate, b-hydroxybutyrate
● b-hydroxybutyrate, Acetoacetate
Correct Answer: Insulin, Glucagon
A deficiency in pyruvate carboxylase activity results in decreased intracellular oxaloacetate and
decreased gluconeogenesis. This can result in a metabolic state which includes _____ and _____.
● Hyperglycemia, Hyperketosis
● Hyperglycemia, Hypoketosis
● Hypoglycemia, Hypoketosis
● Hypoglycemia, Hyperketosis
Correct Answer: Hypoglycemia, Hyperketosis
A carnitine deficiency or an MCAD deficiency, among others, can result in a metabolic state
which includes _____ and _____.
● Hyperglycemia, Hyperketosis
● Hyperglycemia, Hypoketosis
● Hypoglycemia, Hypoketosis
● Hypoglycemia, Hyperketosis
Correct Answer: Hypoglycemia, Hypoketosis
Diabetic ketoacidosis can result in a metabolic state which includes _____ and _____.
● Hyperglycemia, Hyperketosis
● Hyperglycemia, Hypoketosis
● Hypoglycemia, Hypoketosis
● Hypoglycemia, Hyperketosis
Correct Answer: Hyperglycemia, Hyperketosis
Diabetic ketoacidosis can be exacerbated by tissue hypoxia and infections. Typical symptoms do
not include:
● Nausea and vomiting
● Abdominal pain
● Kussmaul breathing
● Hypoglycemia
● Hypotension
Correct Answer: Hypoglycemia
Ketosis has both pathogenic and physiological causes. Which of the following is considered to
be a physiological cause of ketosis? (Select all that apply.)
● Type-1 Diabetes
● Chronic alcoholism
● Enzyme deficiencies
● High fat diet
● Starvation
Correct Answer: High fat diet and Starvation
OLM #4 - Physiology
Which of the following segments of the circulatory system has the lowest velocity of blood
flow?
● Aorta
● Arteries
● Capillaries
● Veins
Answer: Capillaries. The velocity of blood flow within each segment of the circulatory system is
inversely proportional to the total cross-sectional area of the segment. Because the capillaries
have the highest total cross-sectional area of all circulatory segments, it has the lowest velocity
of blood flow.
Which part of the circulation has the highest compliance?
● Capillaries
● Large arteries
● Veins
● Aorta
● Small arteries
Answer: Veins. The vascular compliance is proportional to the vascular distensibility and the
vascular volume of any given segment of the circulation. The compliance of a systemic vein is
24 times that of its corresponding artery because it is about 8 times as distensible and has a
volume about 3 times as great.
Which of the following vessels has the greatest total cross-sectional area in the circulatory
system?
● Aorta
● Small arteries
● Capillaries
● Venules
● Vena cava
Answer: Capillaries. The capillaries have the largest total cross-sectional area of all vessels of the
circulatory system. The venules also have a relatively large total cross-sectional area but not as
great as the capillaries, which explains the large storage of blood in the venous system compared
with that in the arterial system.
A 60-year-old man has a mean arterial blood pressure of 130 mm Hg, a heart rate of 78
beats/min, a right atrial pressure of 0 mm Hg, and a cardiac output of 3.5 L/min. He also has a
pulse pressure of 35 mm Hg and a hematocrit of 40. What is the approximate total peripheral
vascular resistance in this man?
● 17 mm Hg/l/min
● 1.3 mm Hg/l/min
● 13 mm Hg/l/min
● 27 mm Hg/l/min
● 37 mm Hg/l/min
Answer: 37 mm Hg/l/min. Total peripheral vascular resistance = (arterial pressure − right atrial
pressure) ÷ cardiac output. In this example, total peripheral vascular resistance = 130 mm Hg ÷
3.5 l/min, or approximately 37 mm Hg/l/min.
A 50-year-old woman has a renal plasma flow of 600 ml/min and hematocrit of 50. Her arterial
pressure is 125 mm Hg and renal venous pressure is 5 mm Hg. What is the total renal vascular
resistance (in mm Hg/ml/min) in this woman?
● 0.05
● 0.10
● 0.50
● 1.00
● 1.50
Answer: 0.10. Vascular resistance is equal to arterial pressure divided by blood flow. In this
example, arterial pressure is 125 mm Hg, venous pressure is 5 mm Hg, and blood flow is 1200
ml/min (plasma flow/hematocrit). Thus, vascular resistance is equal to 120 divided by (600/.50
or 1200), or 0.10 mm Hg/ml/min.
In control conditions, flow through a blood vessel is 100 ml/min under a pressure gradient of 50
mm Hg. What would be the approximate flow through the vessel after increasing the vessel
diameter to four times normal, assuming that the pressure gradient was maintained at 50 mm Hg?
● 300 ml/min
● 1600 ml/min
● 1000 ml/min
● 16,000 ml/min
● 25,600 ml/min
Answer: E. 25,600 ml/min. According to Poiseuille’s law, flow through a vessel increases in
proportion to the fourth power of the radius. A 4-fold increase in vessel diameter (or radius)
would increase 4 to the fourth power, or 256 times normal. Thus, flow through the vessel after
increasing the vessel 4 times normal would increase from 100 to 25,600 ml/min.
An increase in which of the following would be expected to decrease blood flow in a vessel?
● Pressure gradient across the vessel
● Radius of the vessel
● Plasma colloid osmotic pressure
● Viscosity of the blood
● Plasma sodium concentration
Answer: Viscosity of the blood. The rate of blood flow is directly proportional to the fourth
power of the vessel radius and to the pressure gradient across the vessel. In contrast, the rate of
blood flow is inversely proportional to the viscosity of the blood. Thus, an increase in blood
viscosity would decrease blood flow in a vessel.
The tendency for turbulent flow is greatest in which of the following?
● Arterioles
● Capillaries
● Small arterioles
● Aorta
Answer: Aorta. The tendency for turbulent flow occurs at vascular sites where the velocity of
blood flow is high. The aorta has the highest velocity of blood flow.
Which component of the circulatory system contains the largest percentage of the total blood
volume?
● Arteries
● Capillaries
● Veins
● Pulmonary circulation
● Heart
Answer: Veins. The percentage of total blood volume in the veins is approximately 64%
Which one of the following compensatory physiological changes would be expected to occur in a
person who stands up from a supine position?
● Increased parasympathetic nerve activity
● Increased sympathetic nerve activity
● Decreased heart rate
● Decreased heart contractility
Answer: Increased sympathetic nerve activity. Moving from a supine to a standing position
would tend to cause pooling of blood in the lower extremities and a reduction in arterial pressure.
Decreased arterial pressure activates the baroreceptor reflex which then leads to increased
sympathetic nervous system activity, decreased parasympathetic activity, increased heart rate,
and increased cardiac contractility.
What would tend to increase a person's pulse pressure?
● Decreased stroke volume
● Increased arterial compliance
● Hemorrhage
● Patent ductus
● Decreased venous return
Answer: Patent ductus. The two main factors that affect pulse pressure are stroke volume and
arterial compliance. Decreases in stroke volume decrease pulse pressure, and an increase in
arterial compliance decreases pulse pressure. Hemorrhage and decreased venous return would
decrease stroke volume and pulse pressure. In patients with patent ductus, stroke volume and
pulse pressure are increased as a result of shunting of blood from the aorta to the pulmonary
artery.
A 24-year-old woman delivers a 6-lb, 8-oz baby girl. The newborn is diagnosed as having patent
ductus arteriosus. Which set of changes would be expected in this baby?
Pulse Pressure Stroke Volume Systolic Pressure

A ↑ ↑ ↑

B ↑ ↓ ↑

C ↑ ↓ ↓

D ↑ ↑ ↓

E ↓ ↓ ↓

F ↓ ↑ ↓

G ↓ ↑ ↑
H ↓ ↓ ↑
Answer: A. In patent ductus arteriosus, a large quantity of the blood pumped into the aorta by the
left ventricle immediately flows backward into the pulmonary artery and then into the lung and
left atrium. The shunting of blood from the aorta results in a low diastolic pressure, while the
increased inflow of blood into the left atrium and ventricle increases stroke volume and systolic
pressure. The combined increase in systolic pressure and decrease in diastolic pressure results in
an increase in pulse pressure.
A 60-year-old man visits his family practitioner for an annual examination. He has a mean blood
pressure of 130 mm Hg and a heart rate of 78 beats/min. His plasma cholesterol level is in the
upper 25th percentile, and he is diagnosed as having atherosclerosis. Which set of changes would
be expected in this patient?
Pulse Pressure Arterial Compliance Systolic Pressure

A ↑ ↑ ↑

B ↑ ↓ ↑

C ↑ ↓ ↓

D ↑ ↑ ↓

E ↓ ↓ ↓

F ↓ ↑ ↓

G ↓ ↑ ↑

H ↓ ↓ ↑
Answer: B. A person with atherosclerosis would be expected to have decreased arterial
compliance. The decrease in arterial compliance would lead to an increase in systolic pressure
and pulse pressure.
A healthy 60-year-old woman with a 10-year history of hypertension stands up from a supine
position. Which set of cardiovascular changes is most likely to occur in response to standing up
from a supine position?
Sympathetic Nerve Activity Cardiac Contractility Heart Rate

A ↑ ↑ ↑

B ↑ ↑ ↓

C ↑ ↓ ↓

D ↑ ↓ ↑

E ↓ ↓ ↓
F ↓ ↓ ↑

G ↓ ↑ ↑

H ↓ ↑ ↓
Answer: A. Moving from a supine to a standing position causes an acute fall in arterial pressure
that is sensed by arterial baroreceptors located in the carotid bifurcation and aortic arch.
Activation of the arterial baroreceptors leads to an increase in sympathetic outflow to the heart
and peripheral vasculature and a decrease in parasympathetic outflow to the heart. The increase
in sympathetic activity to peripheral vessels results in an increase in total peripheral resistance.
The increase in sympathetic activity to the heart results in an increase in heart rate and strength
of contraction. The decrease in parasympathetic outflow to the heart also contributes to the
increase in heart rate.
A 60-year-old woman has experienced dizziness for the past 6 months when getting out of bed in
the morning and when standing up. Her mean arterial pressure is 130/90 mm Hg while lying
down and 95/60 while sitting. Which set of physiological changes would be expected in response
to moving from a supine to an upright position?
Parasympathetic Nerve Activity Plasma Renin Activity Sympathetic Activity

A ↑ ↑ ↑

B ↑ ↓ ↑

C ↑ ↓ ↓

D ↑ ↑ ↓

E ↓ ↓ ↓

F ↓ ↑ ↓

G ↓ ↑ ↑

H ↓ ↓ ↑
Answer: G. Moving from a supine to a standing position causes an acute fall in arterial pressure
that is sensed by arterial baroreceptors located in the carotid sinuses and aortic arch. Activation
of the baroreceptors results in a decrease in parasympathetic activity (or vagal tone) and an
increase in sympathetic activity, which leads to an increase in plasma renin activity (or renin
release).
While participating in a cardiovascular physiology laboratory, a medical student isolates an
animal's carotid artery proximal to the carotid bifurcation and partially constricts the artery with
a tie around the vessel. Which set of changes would be expected to occur in response to
constriction of the carotid artery?
Heart Rate Parasympathetic Nerve Activity Total Peripheral Resistance
A ↑ ↑ ↑

B ↑ ↑ ↓

C ↑ ↓ ↓

D ↑ ↓ ↑

E ↓ ↓ ↓

F ↓ ↓ ↑

G ↓ ↑ ↑

H ↓ ↑ ↓
Answer: D. Constriction of the carotid artery decreases blood pressure at the level of the carotid
sinus. A decrease in carotid sinus pressure leads to a decrease in carotid sinus nerve impulses to
the vasomotor center, which in turn leads to enhanced sympathetic nervous activity and
decreased parasympathetic nerve activity. The increase in sympathetic nerve activity results in
peripheral vasoconstriction and an increase in total peripheral resistance and heart rate. The
decreased parasympathetic nerve activity to the heart would also contribute to the increase in
heart rate.
A 22-year-old man enters the hospital emergency department after severing a major artery in a
motorcycle accident. It is estimated that he has lost approximately 700 ml of blood. His blood
pressure is 90/55 mm Hg. Which set of changes would be expected in response to hemorrhage in
this man?
Heart Rate Parasympathetic Nerve Activity Plasma Renin Activity

A ↑ ↑ ↑

B ↑ ↓ ↑

C ↑ ↓ ↓

D ↑ ↑ ↓

E ↓ ↓ ↓

F ↓ ↑ ↓

G ↓ ↑ ↑

H ↓ ↓ ↑
Answer: B. The arterial baroreceptors are activated in response to a fall in arterial pressure.
During hemorrhage, the fall in arterial pressure at the level of the baroreceptors results in
enhanced sympathetic outflow from the vasomotor center and a decrease in parasympathetic
nerve activity. The increase in sympathetic nerve activity leads to constriction of peripheral
blood vessels, increased total peripheral resistance, and plasma renin activity, angiotensin II and
a return of blood pressure toward normal. The decrease in parasympathetic nerve activity and
sympathetic outflow would result in an increase in heart rate.
A 55-year-old man with a history of normal health visits his physician for a checkup. The
physical examination reveals that his blood pressure is 170/98 mm Hg. Further tests indicate that
he has renovascular hypertension as a result of stenosis in the left kidney. Which set of findings
would be expected in this man with renovascular hypertension?
Total Peripheral Plasma Renin Plasma Aldosterone
Resistance Activity Concentration

A ↑ ↑ ↑

B ↑ ↓ ↑

C ↑ ↓ ↓

D ↑ ↑ ↓

E ↓ ↓ ↓

F ↓ ↑ ↓

G ↓ ↑ ↑

H ↓ ↓ ↑
Answer: A. Stenosis of one kidney results in the release of renin and the formation of angiotensin
II from the affected kidney. Angiotensin II stimulates aldosterone production and increases total
peripheral resistance by constricting most of the blood vessels in the body.
An ACE inhibitor is administered to a 65-year-old man with a 20-year history of hypertension.
The drug lowered his arterial pressure and increased his plasma levels of renin. Which
mechanism would best explain the decrease in arterial pressure?
● Inhibition of angiotensin I
● Decreased conversion of angiotensinogen to angiotensin I
● Decreased plasma levels of bradykinin
● Decreased plasma levels of prostacyclin
● Decreased formation of angiotensin II
Answer: Decreased formation of angiotensin II. The conversion of angiotensin I to angiotensin II
is catalyzed by a converting enzyme that is present in the endothelium of the lung vessels and in
the kidneys. The converting enzyme also serves as a kininase that degrades bradykinin. Thus, a
converting enzyme inhibitor not only decreases the formation of angiotensin II but also inhibits
kininases and the breakdown of bradykinin. Angiotensin II is a vasoconstrictor and a powerful
sodium-retaining hormone. While plasma bradykinin increases, the major cause for the decrease
in arterial pressure in response to an ACE inhibitor is the decrease in formation of angiotensin II.
OLM #5 - Anatomy
A 35-year-old male driver was involved in a car accident with air-bag deployment. Radiography
reveals costochondral joint separation for both the fourth and fifth ribs on the left. While still in
the emergency department, he develops dyspnea (difficulty breathing), hypotension, cyanosis,
and neck vein distension. These classic signs of cardiac tamponade result from the accumulation
of fluid in the:
● Superior mediastinum
● Potential space between visceral serous pericardium and myocardium
● Potential space between the parietal and visceral serous pericardium
● Costomediastinal recess
● Potential space between the fibrous and parietal serous pericardium
Correct answer: Potential space between the parietal and visceral serous pericardium
A 38-year-old woman presents with pain to the left side of the neck, extending laterally to the
left clavicle for an hour or two after mild physical activity. A physical examination revealed a
slight temperature and friction sound over the middle of the thorax. Examination of her neck and
shoulder region suggests that the pain may be referred pain since manipulation of the left
shoulder and neck does not reproduce it. Which one of the following nerves is responsible for
referred pain in this woman?
● Intercostal nerve
● Intercostobrachial nerve
● Cardiac plexus
● Phrenic nerve
● Vagus nerve
Correct answer: Phrenic nerve
During a bar fight, a 23-year-old man’s chest is punctured to the left of his sternum with an ice
pick. He is brought to the emergency department because he is feeling faint and unable to walk
due to weakness. He is muffled and his blood pressure is low despite a very rapid pulse. His
lungs sound normal. His jugular veins are becoming more and more distended. How would one
use a 3-inch, 19-gauge needle to help the bar fight victim?
● Insert it just under the left tip of the xiphoid process to remove blood from the
pericardial cavity
● Insert it at the second intercostal space on the left side of the sternum to inject
nitroglycerine to increase the strength of cardiac contractions
● Insert it at the ninth intercostal space at the left midclavicular line in an effort to remove
blood from the pleural cavity
● Insert it at the fourth intercostal space on the right side to remove blood from the right
pulmonary artery
● Insert it just under the left clavicle to remove blood from the right cephalic vein
Correct answer: Insert it just under the left tip of the xiphoid process to remove blood from the
pericardial cavity
A patient involved in an automobile accident presents with a sharp object puncture of the middle
of the sternum at about the level of the 4th or 5th costal cartilage. If the object also penetrated the
pericardium and heart wall, which heart chamber would most likely be damaged?
● Left atrium
● Left ventricle
● Right atrium
● Right ventricle
Correct answer: Right ventricle
A 23-year-old male injured in an industrial explosion was found to have multiple small metal
fragments in his thoracic cavity. Since the pericardium was torn inferiorly, the surgeon began to
explore for fragments in the pericardial sac. Slipping her hand under the heart apex, she slid her
fingers upward and to the right within the sac until they were stopped by the cul-de-sac formed
by the pericardial reflection near the base of the heart. Her fingertips were then in the:
● Coronary sinus
● Coronary sulcus
● Costomediastinal recess
● Oblique sinus
● Transverse sinus
Correct answer: Oblique sinus
Which of the following chambers of the heart is the most closely applied to the esophagus?
● Left atrium
● Left ventricle
● Right atrium
● Right ventricle
● Right auricle
Correct answer: Left atrium
What structures in fetal circulation play a role in shunting blood away from the lungs? Select all
that apply:
● Ductus arteriosus
● Ductus venosus
● Umbilical artery
● Foramen ovale
● Umbilical vein
Correct answers: Ductus arteriosus & Foramen ovale
Which statement below accurately describes the role of the ductus arteriosus?
● The ductus arteriosus helps to connect the umbilical artery to the inferior vena cava
● The ductus arteriosus is found between the right and left atrium
● In fetal circulation, the pulmonary artery and aorta are connected via the ductus
arteriosus
● The ductus arteriosus only carries oxygenated blood from the left side of the heart to the
right side
Correct answer: In fetal circulation, the pulmonary artery and aorta are connected via the ductus
arteriosus
Fill in the blanks: In fetal circulation the umbilical artery carries ___________, while the
umbilical vein carries ___________.
● Deoxygenated blood, oxygenated blood
● Deoxygenated blood, deoxygenated blood
● Oxygenated blood, deoxygenated blood
● Oxygenated blood, oxygenated blood
● None of the above is correct
Correct answer: Oxygenated blood, deoxygenated blood
ANATOMY LAB
Case 1 Questions
A 52-year-old woman is visiting a plastic surgeon specialized on mammoplasty complaining of
neck pain, back pain, and numbness or weakness due to the weight of her breasts. After
thoughtful pre-operative examination, the patient undergoes breast reduction surgery.To preserve
the sensation around her nipples, which of the following nerves should surgeon keep intact in
this procedure?
● Anterior branch of the 2nd intercostal nerve
● Lateral branch of the 5th intercostal nerve
● Anterior branch of the 3rd intercostal nerve
● Lateral branch of the 4th intercostal nerve
● Anterior branch of the 6th intercostal nerve
Correct answer: Lateral branch of the 4th intercostal nerve
The patient is concerned that after the operation on her breast it will detach from her thorax and
slide freely as she moves. To come your patient down, you explain that the mammary gland is
firmly attached to the underlying pectoral fascia. Which of the following structures attach the
mammary gland to the pectoral fascia?
● Lactiferous ducts
● Lactiferous sinuses
● Suspensory ligaments
● Superficial fascia
● Skin
Correct answer: Suspensory ligaments
During the cosmetic procedure on the breast, a dense fibroid mass of 10 to 17 mm was
accidentally found in the lateral lower quadrant of the breast. The mass was dissected and sent to
the laboratory for microscopic analysis. What else should be dissected and taken to the lab to
diagnose the pathology?
● A piece of the adjusted lobule of the mammary gland
● The lactiferous sinus draining this area
● The nearest pectoral lymph node
● A piece of skin overlying the mass
● All axillary lymph nodes
Correct answer: The nearest pectoral lymph node
Case 2
A 46-year-old man is visiting his family doctor due to acute pain under his scapula referring all
around the thorax. The onsets of pain are initiated by movements and get easier at rest. The
physical examination and labs are normal. The X-Ray of his thorax shows no respiratory
pathology. The doctor decided to perform the intercostal block to relieve the patient’s pain. At
which of the following locations should he inject the anesthetic?
● Midaxillary line, 0.5 cm past the inferior border of the 5th rib
● Paravertebral line (6-8 cm lateral to the spinous processes), 0.5 cm past the inferior
border of the 7th rib
● Scapular line, 0.5 cm past the superior border of the 6th rib
● Midclavicular line, 0.5 cm past the superior border of the 7th rib
● Parasternal line, 0.5 cm past the inferior border of the 4th rib
Correct answer: Paravertebral line (6-8 cm lateral to the spinous processes), 0.5 cm past the
inferior border of the 7th rib
Which of the following structures will be reached in this procedure?
Correct Answer: E
OLM #6 - MCB
Fatty Acid synthesis follows a precise biochemical pathway including an ordered set of four
steps. Which of the following lists the correct order of the reaction steps required for fatty acid
synthesis?
● Reduction, Dehydration, Reduction, Condensation
● Reduction, Reduction, Dehydration, Condensation
● Reduction, Reduction, Condensation, Dehydration
● Condensation, Reduction, Dehydration, Reduction
● Condensation, Reduction, Reduction, Dehydration
● Condensation, Dehydration, Reduction, Reduction
Correct Answer: Condensation, Reduction, Dehydration, Reduction
While the enzyme, Isocitrate Dehydrogenase, is not directly involved in Fatty Acid biosynthesis,
regulation of Fatty Acid synthesis does occur through this enzyme. Isocitrate dehydrogenase
plays a critical role in _________. It is inhibited by _______. When Isocitrate dehydrogenase
is down regulated, ________ accumulates and increase levels of this metabolic intermediate are
required for the activation and substrate production for fatty acid synthesis.
● Glycolysis, high levels of ADP and FADH2, a-ketoglutarate
● Glycolysis, high levels of AMP and NADH, a-ketoglutarate
● Glycolysis, high levels of ATP and NADH, citrate
● The Citric Acid Cycle, high levels of ADP and FADH2, a-ketoglutarate
● The Citric Acid Cycle, high levels of ATP and FADH2, a-ketoglutarate
● The Citric Acid Cycle, high levels of ATP and NADH, a-ketoglutarate
● The Citric Acid Cycle, high levels of ATP and NADH, citrate
Correct Answer: The Citric Acid Cycle, high levels of ATP and NADH, citrate
Only one molecule of acetyl-CoA is used directly in the Fatty Acid synthesis reactions carried
out by Fatty Acid Synthase. However, the two carbon units added to the elongating Fatty acyl at
each step are the result of a ___ reaction which converts the 2C ___ to the 3C ___ for use in the
elongation steps of Palmitate/Palmitic acid synthesis.
● Reduction, acetate, malonate
● Dehydration, acetate, malonate
● Condensation, acetate, malonate
● Carboxylation, acetate, malonate
● Reduction, acetyl-CoA, malonyl-CoA
● Dehydration, acetyl-CoA, malonyl-CoA
● Condensation, acetyl-CoA, malonyl-CoA
● Carboxylation, acetyl-CoA, malonyl-CoA
Correct Answer: Carboxylation, acetyl-CoA, malonyl-CoA
The enzyme _____, carries out the reaction described in the previous question. The enzyme
exists within the cytosol of the cell. It is active in the ____ form and inactive in the _____ form.
● Citrate lyase, monomeric, dimeric
● Pyruvate carboxylase, dimeric, monomeric
● Isocitrate dehydrogenase, dimeric, monomeric
● Acetyl-CoA Carboxylase, dimeric, monomeric
● Acetyl-CoA Carboxylase, dimeric, polymeric
● Citrate lyase, dimeric, polymeric
● Pyruvate carboxylase, dimeric, polymeric
● Isocitrate dehydrogenase, dimeric, polymeric
● Acetyl-CoA Carboxylase, polymeric, dimeric
● Citrate lyase, polymeric, dimeric
● Pyruvate carboxylase, polymeric, dimeric
● Isocitrate dehydrogenase, polymeric, dimeric
Correct Answer: Acetyl-CoA Carboxylase, polymeric, dimeric
One level of regulation insulin plays in the regulation of Fatty Acid synthesis is at the level of
gene regulation. Increased levels of insulin activate _____ which results in the activation of ___
genes under the control of the ____ promotor element leading to increase Fatty Acid Synthesis.
● Forkhead transcription factor, FA oxidation, IRE
● SREBP-1 transcription factor, Lipogenic genes, IRE
● ChREBP transcription factor, Lipogenic genes, CarRE
● Forkhead transcription factor, Lipogenic genes, SRE
● SREBP-1 transcription factor, Lipogenic genes, SRE
● ChREBP transcription factor, FA oxidation, CarRE
Correct Answer: SREBP-1 transcription factor, Lipogenic genes, SRE
Acetyl-CoA carboxylase activity is regulated by phosphorylation levels. In the well-fed state,
insulin levels in the plasma are high. This in turn ____ the dephosphorylation of
5’-AMP-Activated Protein Kinase (AMPK) converting AMPK to the _____. This results in ____
phosphorylation levels of Acetyl-CoA carboxylase leading to its ______.
● increases, active form / decreased, activation
● increases, active form / increased, activation
● increases, inactive form / decreased, activation
● decreases, active form / decreased, activation
● decreases, active form / increased, activation
● decreases, inactive form / decreased, activation
Correct Answer: increases, inactive form / decreased, activation
The initial step in the FA synthesis reactions carried out by FA Synthase involves the attachment
of a molecule of ___ to the ___ of FA Synthase by the enzymatic activity of the ___ domain of
the protein. In the subsequent step, that group is transferred to the sulfhydryl group on the ___.
● Malonyl-CoA, ACP-site, Acetyl Transacylase, Cysteine residue
● Acetyl-CoA, ACP-site, Acetyl Transacylase, Cysteine residue
● Fatty acyl-CoA, ACP-site, Acetyl Transacylase, Cysteine residue
● Malonyl-CoA, Cysteine residue, Malonyl Transacylase, ACP-site
● Acetyl-CoA, Cysteine residue, Malonyl Transacylase, ACP-site
● Fatty acyl-CoA, Cysteine residue, Malonyl Transacylase, ACP-site
Correct Answer: Acetyl-CoA, ACP-site, Acetyl Transacylase, Cysteine residue
Following each condensation step in the FA synthesis process carried out by the Ketoacyl
Synthase domain of FA Synthase, the resulting Ketoacyl-group is first converted to a
b-hydroxyacyl-group and subsequently to an enoyl-acyl group. In the correct order, the two
enzymatic domains which carry out this process are:
● Dehydratase, Enoyl Reductase
● Ketoacyl Reductase, Enoyl Reductase
● Ketoacyl Reductase, Dehydratase
● Ketoacyl Reductase, Thioesterase
● Thioesterase, Dehydratase
Correct Answer: Ketoacyl Reductase, Dehydratase
Following the necessary chain-extending cycles, the 16 carbon Palmitic acid or Palmitate is
released from the ACP-site on the FA Synthase enzyme by the enzymatic domain with the ____
activity.
● Dehydratase
● Ketoacyl Reductase
● Enoyl Reductase
● Thioesterase
● Ketoacyl Synthase
● Malonyl Transacylase
● Acetyl Transacylase
Correct Answer: Thioesterase
Both the elongation and desaturation of Fatty Acids occurs in the ____.
● Peroxisome
● Lysosome
● Mitochondria
● Cytosol
● RER
● SER
● Golgi
Correct Answer: SER
The initial reducing power used by FA Desaturases to introduce points of desaturation in fatty
acids comes from NADPH. The electrons from NAPDH are first passed to ___ functioning as a
coenzyme for Cytochrome B5 reductase. From there the electrons are transferred to the ___
prosthetic group of Cytochrome B5 which provides the electrons used in formation of the double
bond in the newly formed point of unsaturation.
● FAD, Heme
● FADH2 , NADH
● NAD+, Heme
● NADH, FADH2
● CoEnzymeQ, Heme
Correct Answer: FAD, Heme
In order to produce triacylglycerides from fatty acids, glycerol-3-phosphate (G3P) is required.
_____ from glycolysis can be directly converted to G3P in both the cells of the liver and adipose
tissues via the enzyme _______. However, only the liver is able to directly phosphorylate
glycerol to G3P through the enzyme _____.
● a-ketoglytrate, glycerol-3-phosphate dehydrogenase, Glycerol kinase
● Dihydroxyacetone phosphate, glycerol-3-phosphate dehydrogenase, Glycerol kinase
● Phosphoenolpyruvate, glycerol-3-phosphate dehydrogenase, Glycerol kinase
● Glyceraldehyde-3-phophate, glycerol-3-phosphate dehydrogenase, Glycerol kinase
● Dihydroxyacetone phosphate, Glycerol kinase, glycerol-3-phosphate dehydrogenase
Correct Answer: Dihydroxyacetone phosphate, glycerol-3-phosphate dehydrogenase, Glycerol
kinase
OLM #7 - Physiology
A healthy 27-year-old female medical student runs a 5K race. Which set of physiological
changes is most likely to occur in this woman's skeletal muscles during the race?
Arteriole Resistance Tissue pH Tissue Carbon Dioxide
Concentration

A ↑ ↑ ↑

B ↑ ↑ ↓

C ↑ ↓ ↓

D ↑ ↓ ↑

E ↓ ↓ ↓

F ↓ ↓ ↑

G ↓ ↑ ↑

H ↓ ↑ ↓

Answer: F. The increase in local metabolism during exercise increases carbon dioxide production
and decreases tissue oxygen concentration and tissue pH. The decrease in tissue oxygen
concentration, tissue pH, and increase in carbon dioxide production increase arteriolar diameter
(decrease arteriole resistance) and increase vascular conductance and blood flow to skeletal
muscles.
An increase in shear stress in a blood vessel results in which change?
● Decreased endothelin production
● Decreased cyclic guanosine monophosphate production
● Increased nitric oxide release
● Increased renin production
● Decreased prostacyclin production
Answer: Increased nitric oxide release. An increase in shear stress in blood vessels is one of the
major stimuli for the release of nitric oxide by endothelial cells. Nitric oxide increases blood
flow by increasing cyclic guanosine monophosphate.
Autoregulation of tissue blood flow in response to an increase in arterial pressure occurs as a
result of which of the following?
● Decrease in vascular resistance
● Initial decrease in vascular wall tension
● Excess delivery of nutrients such as oxygen to the tissues
● Decrease in tissue metabolism
Answer: Excess delivery of nutrients such as oxygen to the tissues. An increase in perfusion
pressure to a tissue results in excessive delivery of nutrients such as oxygen to a tissue. The
increase in tissue oxygen concentration constricts arterioles and returns blood flow and nutrient
delivery toward normal levels.
A mother informs the pediatrician that she wants to hold down her daughter during venipuncture
since the girl is extremely fearful of needles. When the girl sees the venipuncture needle and
becomes anxious, which of the following circulatory beds will respond with the largest percent
decrease in blood flow compared to pre-stimulation?
● Cerebral
● Coronary
● Cutaneous
● Pulmonary
● Skeletal muscle
Answer: Cutaneous. With no ambient temperature change, blood will be redirected during a
fight-or-flight response (triggered by extreme fear) from the skin to vital organs that are needed
for fighting and running.
Cognitive stimuli such as reading, problem solving, and talking all result in significant increases
in cerebral blood flow. Which set of changes in cerebral tissue concentrations is the most likely
explanation for the increase in cerebral blood flow?
Carbon Dioxide pH Adenosine

A ↑ ↑ ↑

B ↑ ↓ ↑

C ↑ ↓ ↓

D ↑ ↑ ↓

E ↓ ↓ ↓

F ↓ ↑ ↓

G ↓ ↑ ↑

H ↓ ↓ ↑

Answer: B. Cognitive stimuli increase cerebral blood flow by decreasing cerebral vascular
resistance. The diameter of cerebral vessels is increased by various metabolic factors in response
to cognitive stimuli. Metabolic factors that enhance cerebral blood flow include increases in
carbon dioxide, hydrogen ion (decreased pH), and adenosine.
ANATOMY LAB
Match the structures with their labels:

1. Lung
2. Pleural cavity
3. Diaphragm
4. Pericardium
5. Thoracic wall
During the dissection of the mediastinum, you accidentally cut the structure running along the
lateral side of the pericardium just anterior to the root of the lung (see the image below). What is
that structure?

Correct answer: Phrenic nerve


Consider the previous scenario: If such complication happens during the operation on an alive
patient, what complication would you expect?
● Paralysis of the intercostal muscles
● Paralysis of the lavatories costarum
● Paralysis of the transverse thoracic muscle
● Paralysis of the diaphragm
● Paralysis of the abdominal muscles
Correct answer: Paralysis of the diaphragm
During an operation on the heart, the surgeon needs to put a temporal ligature on the pulmonary
trunk and ascending aorta. Through which of the following spaces will the ligature pass?
● Oblique sinus
● Coronary sulcus
● Transverse sinus
● Costodiaphragmatic recess
● Thoracic outlet
Correct answer: Transverse sinus
Match the structures with their labels:

1. Aorta
2. Pulmonary trunk
3. Pulmonary veins
4. IVC
5. SVC
OLM #8 - Anatomy
Coronary angiography done in advance of coronary bypass surgery on a 54-year-old female
reveals that she has a “right dominant heart.” This condition:
● Is a major correlate with coronary artery disease
● Is present in < 10% of female patients
● This indicates that the posterior interventricular artery is derived from the right coronary
artery
● Indicates that the right and left coronary arteries are derived from a common trunk
● Is frequently associated with atrial fibrillation
Correct answer: This indicates that the posterior interventricular artery is derived from the right
coronary artery
Blockage of blood flow in the proximal part of the anterior interventricular artery could deprive a
large area of heart tissue of blood supply unless a substantial retrograde flow into this artery
develops via an important anastomosis with which other artery?
● Circumflex
● Left marginal
● Posterior interventricular
● Right coronary
● Right marginal
Correct answer: Posterior interventricular
Blockage of which of the following arteries would lead to ischemia of the apex of the heart?
● Anterior interventricular (descending)
● Left circumflex
● Posterior interventricular (descending)
● Right marginal
● Right coronary
Correct answer: Anterior interventricular (descending)
While attempting to suture the distal end of a coronary bypass onto the anterior interventricular
artery, the surgeon accidentally passed the needle through the adjacent vein. Which vein was
damaged?
● Anterior cardiac vein
● Coronary sinus
● Great cardiac vein
● Middle cardiac vein
● Small cardiac vein
Correct answer: Great cardiac vein
Coronary arteries supply blood to the myocardium and pacemaker centers of the heart from
branches that come off either the right or left coronary arteries. While some regions of the
myocardium may receive blood from two different coronary arteries, the pacemaker centers of
the heart normally only receive blood from a single coronary artery, but the pattern varies in the
human population. The blood supply to the sinoatrial (SA) and atrioventricular (AV) nodes is
derived from which coronary artery(ies)?
● Both the SA and AV nodes receive blood from the left coronary artery
● Both the SA and AV nodes are supplied by the posterior interventricular artery
● In 60% of the population, the SA node receives its blood from the right coronary artery;
in 80% of the population, the AV node receives its blood from the right coronary artery
via the posterior interventricular artery
● In 60% of the population, the SA node receives its blood from the left coronary artery;
in 80% of the population, the AV node receives its blood from the left coronary artery
via the posterior interventricular artery
● Both SA and AV nodes may be supplied by the right or left coronary arteries equally
(50:50%)
Correct answer: In 60% of the population, the SA node receives its blood from the right coronary
artery; in 80% of the population, the AV node receives its blood from the right coronary artery
via the posterior interventricular artery.
An elderly lady suffers a coronary occlusion and subsequently, it is noted that there is a complete
heart block (that is, the right and left bundles of the conduction system have been damaged). The
artery most likely involved in the:
● Right marginal branch
● Circumflex branch
● Anterior interventricular (Left anterior descending)
● Left marginal branch
● Posterior interventricular (posterior descending)
Correct answer: Anterior interventricular (Left anterior descending)
Which of the following branches of the cardiac plexus are postganglionic sympathetic?
● Superior (cervical) cardiac branches
● Inferior (thoracic) cardiac branches
● Branches of the left recurrent laryngeal n.
● Middle splanchnic cardiac branches
● White rami of T1-T2 spinal nerves
Correct answer: Middle splanchnic cardiac branches
Which of the following nerves does participate in the formation of the superficial part of the
cardiac plexus?
● Left superior splanchnic cardiac n.
● Left middle splanchnic cardiac n.
● Right inferior splanchnic cardiac n.
● Right superior splanchnic cardiac n.
● Right superior cardiac branches of the vagus
Correct answer: Left superior splanchnic cardiac n.
The irritation of the vagus nerve in the neck would lead to one of the following symptoms:
● Increase of the atrioventricular conduction
● Increase in the heart rate
● Increase of the atrial and ventricular contractility
● Constriction of the coronary arteries
● Stimulation of the SA node
Correct answer: Constriction of the coronary arteries
OLM #9 - Physiology
Which of the following capillaries has the highest capillary permeability to plasma albumin?
● Glomerular
● Liver
● Muscle
● Intestinal
● Brain
Answer: Liver. The liver endothelium contains many open pores, or fenestrae that allow
extremely large molecules such as albumin to pass in or out of the liver tissues. Under normal
conditions, very little if any albumin cross the capillary walls of muscle, glomerular, brain, or
intestine.
Which one of the following would tend to increase capillary filtration rate?
● Decreased capillary hydrostatic pressure
● Decreased plasma colloid osmotic pressure
● Decreased interstitial colloid osmotic pressure
● Decreased capillary water permeability
● Increased arteriolar resistance
Answer: Decreased plasma colloid osmotic pressure. Increased capillary hydrostatic
pressure, decreased plasma colloid osmotic pressure, and increased interstitial colloid osmotic
pressure would all promote filtration.
The diameter of a precapillary arteriole is decreased in a muscle vascular bed. Which change in
the microcirculation would be expected?
● Decreased capillary filtration rate
● Increased interstitial volume
● Increased lymph flow
● Increased capillary hydrostatic pressure
● Decreased arteriolar resistance
Answer: Decreased capillary filtration rate. A decrease in the diameter of a precapillary arteriole
increases arteriolar resistance while decreasing vascular conductance and capillary blood flow,
hydrostatic pressure, filtration rate, interstitial volume, and interstitial hydrostatic pressure.
An increase in which of the following tends to decrease capillary filtration rate?
● Capillary hydrostatic pressure
● Plasma colloid osmotic pressure
● Interstitial colloid osmotic pressure
● Venous hydrostatic pressure
● Arteriolar diameter
Answer: Plasma colloid osmotic pressure. An increase in plasma colloid osmotic pressure would
reduce net filtration pressure and capillary filtration rate. Increases in capillary hydrostatic
pressure and interstitial colloid osmotic pressure would also favor capillary filtration. An
increase in venous hydrostatic pressure and arteriolar diameter would tend to increase capillary
hydrostatic pressure and capillary filtration rate.
Which pressure is normally negative in a muscle capillary bed in the lower extremities?
● Plasma colloid osmotic pressure
● Capillary hydrostatic pressure
● Interstitial hydrostatic pressure
● Interstitial colloid osmotic pressure
● Venous hydrostatic pressure
Answer: Interstitial hydrostatic pressure. Interstitial hydrostatic pressure in a muscle capillary
bed is normally negative (−3 mm Hg). Pumping by the lymphatic system is the basic cause of the
negative pressure.
A nitric oxide donor is infused into the brachial artery of a 22-year-old man. Which set of
microcirculatory changes would be expected in the infused arm?
Capillary Hydrostatic Interstitial Lymph Flow
Pressure Hydrostatic Pressure

A ↑ ↑ ↑

B ↑ ↑ ↓

C ↑ ↓ ↓
D ↑ ↓ ↑

E ↓ ↓ ↓

F ↓ ↓ ↑

G ↓ ↑ ↑

H ↓ ↑ ↓

Answer: A. Nitric oxide is a vasodilator that is believed to play a role in regulating blood flow.
Infusion of a nitric oxide donor into the brachial artery would increase arteriolar diameter and
decrease arteriolar resistance. The decrease in arteriolar resistance would also result in an
increase in capillary hydrostatic pressure and filtration rate. The increase in filtration rate leads to
an increase in interstitial hydrostatic pressure and lymph flow.
OLM #10 - MCB
Drag the class of complex lipid to their corresponding structural descriptions below.
● One fatty acid coupled to a sphingosine backbone via an ester linkage, and a polar head
group connected via a phosphodiester bond: Sphingophospholipids
● 3 fatty acids coupled to a glycerol backbone via ester linkages: Triglycerides
● 2 fatty acids coupled to a glycerol backbone by ester linkages, and a polar head group
connected via a phosphodiester bond: Glycerophospholipids
● One fatty acid coupled to a sphingosine backbone via an ester linkage, and a
carbohydrate group connected via an O-linkage: Glycolipids
● 1 fatty acid coupled to a glycerol backbone by an ester linkage, one by an ether linkage,
and a polar head group connected via a phosphodiester bond: Ether glycerolipids
Drag the lipids to indicate whether they are more commonly found in the inner or outer leaflets
of plasma membranes, or in mitochondrial membranes.
Plasma Membrane Inner Leaflet
● Phosphatidylserine
● Phosphatidylethanolamine
● Phosphatidylinositol
Plasma Membrane Outer Leaflet
● Sphingomyelin
● Phosphatidylcholine
Inner Mitochondrial Membrane
● Cardiolipin
Drag the phospholipid to its corresponding function.
● Participates in hypersensitivity and anaphylactic reactions: Platelet Activating Factor
● Decreases permeability of the inner mitochondrial membrane: Cardiolipin
● Promotes formation of a hemostatic plug following vascular injury: Platelet Activating
Factor
● The lipid component of a GPI anchor embedded in a membrane: Phosphatidylinositol
● A major component of lung surfactant: Dipalmitoyl Phosphatidylcholine (DPCC)
● Reservoir of DAG and IP3 second messengers: Phosphatidylinositol
Phospholipids are synthesized by 2 different strategies. Indicate which phospholipids are
synthesized by which strategy, by dragging them to the appropriate column.
Strategy 1: CDP is added to the head group before addition to diacylglycerol
● Phosphatidylcholine
● Phosphatidylethanolamine
● Phosphatidylserine
Strategy 2: CDP is added to diacylglycerol before addition of the head group
● Phosphatidylinositol
● Cardiolipin
● Phosphatidylglycerol
If free choline is limiting, phosphatidylcholine can be synthesized by which of the following
strategies?
● Methylation of phosphatidylserine by SAM
● Methylation of diacylglycerol by SAM
● Methylation of phosphatidylethanolamine by SAM
● Sulfation of phosphatidylethanolamine by PAPS
● Sulfation of phosphatidylserine by PAPS
● Sulfation of phosphatidylserine by PAPS
Correct answer: Methylation of phosphatidylethanolamine by SAM
Phospholipases degrade phospholipids. For each of the main phospholipases, drag them to the
description of their corresponding activity.
● Cleaves a fatty acid coupled to the glycerol backbone at carbon 1: Phospholipase A1
● Cleaves a fatty acid coupled to the glycerol backbone at carbon 2: Phospholipase A2
● Cleaves the head group and phosphate from the glycerol backbone at carbon 3:
Phospholipase C
● Releases arachidonic acid for production of eiconsanoids: Phospholipase A2
● Releases DAG and IP3 as second messengers: Phospholipase C
Drag each of the molecules to their corresponding descriptions.
● Ceramide coupled to a phosphocholine head group: Sphingomyelin
● Ceramide coupled to a sulfated galactose: Sulfatide
● Ceramide coupled to an oligosaccharide chain including NANA: Ganglioside
● A palmitate coupled to a serine: Sphingosine
● Ceramide coupled to a glucose: Glucocerebroside
● Ceramide coupled to an oligosaccharide chain without NANA: Globoside
● Ceramide coupled to a galactose: Galactocerebroside
● A sphingosine coupled to a fatty acid: Ceramide
Individuals with type AB blood… (Choose all correct answers)
● Can donate blood to individuals with type O, A or B blood
● Can receive blood transfusions from individuals with any of type O, A or B blood
● Express only the H antigen on the surface of their RBCs
● Express both the A and B antigens on the surface of the RBCs
● Produce antibodies against both the A and B antigens
● Don’t produce antibodies against either A or B antigens
Correct answers:
● Can receive blood transfusions from individuals with any of type O, A or B blood
● Express both the A and B antigens on the surface of the RBCs
● Don’t produce antibodies against either A or B antigens
Which of the following are common lipid components of the myelin sheath? (Choose all correct
answers).
● Glucocerebroside
● Sphingomyelin
● Sulfatide
● Cardiolipin
● PAPS
● Platelet Activating Factor
● Galactocerebroside
● SAM
● Ethanolamine plasmalogen
Correct answers:
● Sphingomyelin
● Sulfatide
● Galactocerebroside
● Ethanolamine plasmalogen
Drag the disease to the corresponding genetic defect.
● Deficiency of the hexosaminidase A beta subunit: Sandhoff
● Deficiency of glucocerebrosidase: Gaucher
● Deficiency of beta-galactosidase: Krabbe
● Deficiency of the hexosaminidase A alpha subunit: Tay-Sachs
● Deficiency of alpha-galactosidase: Fabry
● Loss-of-function of the GM2A gene: Activator deficiency
● Deficiency of arylsulfatase: Metachromatic leukodystrophy
For each of the following clinical or histological findings, drag the corresponding disease for
which they are best associated.
● Vacuolated lymphocytes: Tay Sachs/Sandhoff
● Maltese crosses in urine: Fabry
● Onion skin/whorled lysosomes: Tay Sachs/Sandhoff
● Crumpled tissue paper cytoplasm: Gaucher
● Cherry-red spot in the retina: Niemann-Pick/Tay Sachs/Sandhoff
● Zebra bodies: Niemann-Pick/Fabry
● Ballooned Neurons: Tay Sachs/Sandhoff
● Deficient blood clotting: Gaucher
● Angiokeratoma: Fabry
Hepatosplenomegaly is NOT observed in which of the following diseases? (Choose all correct)
● Niemann-Pick
● Tay Sachs / Sandhoff
● Gaucher
● Fabry
● Krabbe
● Metachromatic leukodystrophy
Correct answers:
● Tay Sachs / Sandhoff
● Fabry
● Krabbe
● Metachromatic leukodystrophy
Which of the following diseases is X-Linked?
● Niemann-Pick
● Tay Sachs / Sandhoff
● Gaucher
● Fabry
● Krabbe
● Metachromatic leukodystrophy
Correct answer: Fabry
OLM #11 - Anatomy
Imaging of the thorax in an 18-month-old female reveals a diffuse structure in the superior
mediastinum, representing the thymus gland. All of the following also would be found in the
superior mediastinum except:
● Phrenic nerve
● Vagus nerve
● Ascending aorta
● Left brachiocephalic vein
● Brachiocephalic artery
Correct answer: Ascending aorta
A transverse CAT scan at the level of the intervertebral disc between T4 and T5 in a 29-year-old
male reveals normal anatomy. All of the following might be seen in this scan except:
● Superior vena cava
● Pulmonary valve
● The superior lobe of the right lung
● Arch of the azygos vein
● Arch of aorta
Correct answer: Pulmonary valve
The most likely structure to be compressed by a tumor found above the root of the left lung is
which of the following?
● Arch of the aorta
● Left pulmonary vein
● Pulmonary trunk
● Ascending aorta
● Azygos vein
Correct answer: Arch of the aorta
An 8-year-old boy is found to have a mid-line tumor of the thymus gland that is impinging
posteriorly on a blood vessel. The affected vessel is most likely the:
● Left brachiocephalic vein
● Left pulmonary vein
● Left bronchial vein
● Right pulmonary artery
● Right superior intercostal vein
Correct answer: Left brachiocephalic vein
A tumor of the posterior mediastinum is most likely to compress which of the following
structures?
● Arch of the aorta
● Esophagus
● Inferior vena cava
● Pulmonary trunk
● Trachea
Correct answer: Esophagus
While performing transesophageal echocardiography on a patient, the posterior wall of the
esophagus, immediately behind the left atrium, was punctured from within. The patient
subsequently developed an infection in the space around the esophagus at this point, namely the:
● Anterior mediastinum
● Middle mediastinum
● Posterior mediastinum
● Superior mediastinum
Correct answer: Posterior mediastinum
Since the puncture in the previous question was through the posterior wall of the esophagus, the
doctors were also very concerned about possible damage to a thin-walled vessel just behind the
esophagus and between the azygos vein and aorta, i.e., the:
● Hemiazygos vein
● Left bronchial vein
● Left pulmonary vein
● Superior vena cava
● Thoracic duct
Correct answer: Thoracic duct
OLM #12 - MCB
Which of the following are true of cholesterol? (Choose all correct answers)
● Cholesterol is relatively unstable
● Cholesterol is completely hydrophobic
● Eating a cholesterol-rich diet has a strong effect on plasma cholesterol levels
● A single hydroxyl group makes cholesterol slightly amphipathic
● It consists of 27 carbons with 4 rings and a branched hydrocarbon chain
● At physiological temperatures cholesterol increases membrane fluidity
● At physiological temperatures cholesterol decreases membrane fluidity
● A biochemical catabolic pathway degrades cholesterol
● Cholesterol esters consist of a molecule of cholesterol coupled to a carbohydrate by an
ester linkage
● Cholesterol esters consist of a molecule of cholesterol coupled to a fatty acid by an ester
linkage
● Cholesterol esters are more hydrophobic than free cholesterol
● Hypercholesterolemia can be easily diagnosed by a physical exam
Correct answers:
● A single hydroxyl group makes cholesterol slightly amphipathic
● It consists of 27 carbons with 4 rings and a branched hydrocarbon chain
● At physiological temperatures cholesterol decreases membrane fluidity
● Cholesterol esters consist of a molecule of cholesterol coupled to a fatty acid by an ester
linkage
● Cholesterol esters are more hydrophobic than free cholesterol
Drag the following terms to the correct column to indicate whether they represent influxes or
effluxes to the liver cholesterol pool.
Influx
● De novo synthesis
● Dietary cholesterol
● Cholesterol carried by HDL
Efflux
● Synthesis of bile acids and salts
● Cholesterol carried by VLDL
● Biliary cholesterol
Which of the following are true of cholesterol synthesis? (Choose all correct answers)
● Cholesterol is synthesized in the mitochondria
● Cholesterol is synthesized in the cytoplasm
● Cholesterol is synthesized only in tissues with mitochondria
● Cholesterol is synthesized only in hepatocytes
● The initial substrate for cholesterol synthesis is Acetyl-CoA
● The initial substrate for cholesterol synthesis is pyruvate
● The rate-limiting, regulated enzyme is Thiolase
● The rate-limiting, regulated enzyme is HMG-CoA Reductase
Correct answers:
● Cholesterol is synthesized in the cytoplasm
● Cholesterol is synthesized only in tissues with mitochondria
● The initial substrate for cholesterol synthesis is Acetyl-CoA
● The rate-limiting, regulated enzyme is HMG-CoA Reductase
Which intermediate of cholesterol synthesis represents the product of the rate limiting step?
● HMG-CoA
● Mevalonate
● 5-carbon Isoprenoid units
● Geranyl pyrophosphate
● Farnesyl pyrophosphate
● Lanosterol
● 7-Dehydrocholesterol
Correct answer: Mevalonate
Which intermediate of cholesterol synthesis is a branch point for the biosynthesis of dolichol,
heme A side chains, ubiquinone and prenyl groups on proteins?
● HMG-CoA
● Mevalonate
● 5-carbon Isoprenoid units
● Geranyl pyrophosphate
● Farnesyl pyrophosphate
● Lanosterol
● 7-Dehydrocholesterol
Correct answer: Farnesyl pyrophosphate
Which intermediate of cholesterol synthesis is an immediate precursor for synthesis of vitamin D
in the skin upon exposure to UV light?
● HMG-CoA
● Mevalonate
● 5-carbon Isoprenoid units
● Geranyl pyrophosphate
● Farnesyl pyrophosphate
● Lanosterol
● 7-Dehydrocholesterol
Correct answer: 7-Dehydrocholesterol
Which intermediate of cholesterol synthesis is the first sterol synthesized?
● HMG-CoA
● Mevalonate
● 5-carbon Isoprenoid units
● Geranyl pyrophosphate
● Farnesyl pyrophosphate
● Lanosterol
● 7-Dehydrocholesterol
Correct answer: Lanosterol
Which of the following are true of Smith-Lemili-Opitz Syndrome? (Choose all correct answers)
● Caused by a deficiency of HGM-CoA Reductase
● Caused by a deficiency of 7-dehydrocholesterol reductase
● Inheritance is autosomal recessive
● Inheritance is X-linked recessive
● Cholesterol deficiencies affect Shh signalling leading to developmental abnormalities
● Cholesterol precursors accumulate to pathological levels
● Clinical presentation includes intellectual disability, short stature, prominent jaw and
broad forehead, cleft lip and/or cleft palate
● Clinical presentation includes intellectual disability, polydactyly microcephaly,
congenital malformations of the heart, lungs, kidneys and GI tract
Correct answers:
● Caused by a deficiency of 7-dehydrocholesterol reductase
● Inheritance is autosomal recessive
● Cholesterol deficiencies affect Shh signalling leading to developmental abnormalities
● Cholesterol precursors accumulate to pathological levels
● Clinical presentation includes intellectual disability, polydactyly microcephaly,
congenital malformations of the heart, lungs, kidneys and GI tract
Which of the following occurs in hepatocytes when the intracellular cholesterol levels are high?
(Choose all correct answers)
● ACAT is activated to increase production of cholesterol esters
● ACAT is inhibited to decrease production of cholesterol esters
● HMG-CoA Reductase is activated to increase synthesis of cholesterol
● HMG-CoA Reductase is inhibited to decrease synthesis of cholesterol
● Gene expression of the LDL Receptor is activated
● Gene expression of the LDL Receptor is inhibited
Correct answers:
● ACAT is activated to increase production of cholesterol esters
● HMG-CoA Reductase is inhibited to decrease synthesis of cholesterol
● Gene expression of the LDL Receptor is inhibited
Which of the following is true regarding the regulation of HMG-CoA reductase? (Choose all
correct answers)
● HMG-CoA reductase is activated in the fasting state and inhibited in the well-fed state
● HMG-CoA reductase is active when dephosphorylated and inactive when
phosphorylated
● High levels of AMP will inactivate HMG-CoA reductase
● Insulin signalling activates HMG-CoA reductase by triggering dephosphorylation of the
enzyme
● Glucagon signalling activates HMG-CoA reductase by triggering phosphorylation of the
enzyme
● SREBP activates expression of HMG-CoA reductase when sterol levels are high
● SREBP activates expression of HMG-CoA reductase when sterol levels are low
Correct answers:
● HMG-CoA reductase is active when dephosphorylated and inactive when
phosphorylated
● High levels of AMP will inactivate HMG-CoA reductase
● Insulin signalling activates HMG-CoA reductase by triggering dephosphorylation of the
enzyme
● SREBP activates expression of HMG-CoA reductase when sterol levels are low
Which of the following are true of statins? (Choose all correct answers)
● Statins directly inhibit synthesis of cholesterol in the plasma
● Statins inhibit synthesis of intracellular hepatic cholesterol which leads to increased
clearance of plasma LDL by hepatic LDL-receptors
● Statins promote phosphorylation of HMG-CoA reductase
● Statins are irreversible inhibitors of HMG-CoA reductase
● Statins are structural analogs of HMG-CoA
● Statins inhibit synthesis of components of the electron transport chain
Correct answers:
● Statins inhibit synthesis of intracellular hepatic cholesterol which leads to increased
clearance of plasma LDL by hepatic LDL-receptors
● Statins are structural analogs of HMG-CoA
● Statins inhibit synthesis of components of the electron transport chain
Which of the following are true of bile? (Choose all correct answers)
● Bile salts solubilize the bile
● Bile consists primarily of bile salts, phosphatidylcholine and free cholesterol
● Bile salts and phospholipids in bile are emulsifying agents
● Bile is synthesized in the gall bladder
● Bile is stored and concentrated in the gall bladder
● Bile is necessary for digestion and absorption of carbohydrates
● Bile is necessary for digestion and absorption of lipids
● Bile is necessary for digestion and absorption of B vitamins and vitamin C
● Bile is necessary for digestion and absorption of vitamins A, D, E and K
● Bile provides a mechanism for elimination of cholesterol from the body via the feces
Correct answers:
● Bile consists primarily of bile salts, phosphatidylcholine and free cholesterol
● Bile salts and phospholipids in bile are emulsifying agents
● Bile is stored and concentrated in the gall bladder
● Bile is necessary for digestion and absorption of lipids
● Bile is necessary for digestion and absorption of vitamins A, D, E and K
● Bile provides a mechanism for elimination of cholesterol from the body via the feces
Which of the following are true of bile acids and salts? (Choose all correct answers)
● Bile acids are negatively charged and better emulsifiers than bile salts
● Bile salts are negatively charged and better emulsifiers than bile acids
● Unconjugated bile acids have a lower pKa than the pH of the intestinal lumen so are
primarily deprotonated
● Conjugated bile acids have a lower pKa than the pH of the intestinal lumen so are
primarily deprotonated
● Conjugated bile acids are better emulsifiers than unconjugated bile acids
● Secondary bile acids are better emulsifiers than primary bile acids
● Secondary bile acids are synthesized de novo in the liver
● Secondary bile acids are produced by bacterial action in the intestines
● The vast majority of bile acids in the intestines are excreted in the feces
● The vast majority of bile acids in the intestines are reabsorbed and returned to the liver
via portal circulation
Correct answers:
● Bile salts are negatively charged and better emulsifiers than bile acids
● Conjugated bile acids have a lower pKa than the pH of the intestinal lumen so are
primarily deprotonated
● Conjugated bile acids are better emulsifiers than unconjugated bile acids
● Secondary bile acids are produced by bacterial action in the intestines
● The vast majority of bile acids in the intestines are reabsorbed and returned to the liver
via portal circulation
How does cholestyramine lower plasma cholesterol?
● It directly inhibits synthesis of plasma cholesterol
● It inhibits synthesis of intracellular hepatic cholesterol which leads to increased
clearance of plasma LDL by the LDL-receptor
● It blocks bile acid reabsorption, causing hepatic cholesterol to be diverted to the bile,
which leads to increased clearance of plasma LDL by hepatic LDL-receptors
● It limits absorption of dietary cholesterol and reabsorption of biliary cholesterol, which
leads to increased clearance of plasma LDL by hepatic LDL-receptors
● It inhibits lipolysis and release of fatty acids in adipose, thereby inhibiting VLDL
production in the liver and thus lowering LDL levels
Correct answer: It blocks bile acid reabsorption, causing hepatic cholesterol to be diverted to the
bile, which leads to increased clearance of plasma LDL by hepatic LDL-receptors
How does ezetimibe lower plasma cholesterol?
● It directly inhibits synthesis of plasma cholesterol
● It inhibits synthesis of intracellular hepatic cholesterol which leads to increased
clearance of plasma LDL by the LDL-receptor
● It blocks bile acid reabsorption, causing hepatic cholesterol to be diverted to the bile,
which leads to increased clearance of plasma LDL by hepatic LDL-receptors
● It limits absorption of dietary cholesterol and reabsorption of biliary cholesterol, which
leads to increased clearance of plasma LDL by hepatic LDL-receptors
● It inhibits lipolysis and release of fatty acids in adipose, thereby inhibiting VLDL
production in the liver and thus lowering LDL levels
Correct answer: It limits absorption of dietary cholesterol and reabsorption of biliary cholesterol,
which leads to increased clearance of plasma LDL by hepatic LDL-receptors
Which of the following would lead to an increased risk of cholelithiasis? (Choose all correct
answers)
● Increased production of bile acids/salts
● Decreased transport of phosphatidylcholine to the bile
● Decreased transport of free cholesterol to the bile
● Increased concentration of free cholesterol in the bile
● Decreased concentration of bile acids/salts in the bile
Correct answers:
● Decreased transport of phosphatidylcholine to the bile
● Increased concentration of free cholesterol in the bile
● Decreased concentration of bile acids/salts in the bile
OLM #13 - Histology
Which of the following cell types is predominantly found in the myocardium?
● Epithelial cells
● Mesothelial cells
● Cardiomyocytes
● Smooth muscle cells
● Purkinje fibers
Correct Answer: Cardiomyocytes
Which of the following cell types makes up the outermost portion of the epicardium?
● Purkinje Fibers
● Visceral Serous Pericardium
● Fibrous Pericardium Tissue
● Myocardial cells
● Endothelial cells
Correct Answer: Serous Pericardium cells
Which of the following layers make up the borders of the pericardial cavity?
● Purkinje Fibers and Endothelial Cells
● Parietal and Visceral Fibrous Pericardium
● Inner Serous and Outer Fibrous Pericardium
● Inner Endocardium and Outer Epicardium
● Parietal and Visceral Serous Pericardium
Correct Answer: Parietal and Visceral Serous Pericardium
What type of tissue is the fibrous pericardium composed of?
● Lamina propria
● Mesothelium
● Adipose tissue
● Cardiac muscle
● Endothelium
Correct Answer: Adipose Tissue
What type of tissue is the serous pericardium composed of?
● Lamina propria
● Mesothelium
● Adipose tissue
● Cardiac muscle
● Endothelium
Correct Answer: Mesothelium
What type of tissue makes up the innermost layer of the endocardium?
● Lamina propria
● Mesothelium
● Adipose tissue
● Cardiac muscle
● Endothelium
Correct Answer: Endothelium
Which of the following best explains why the cardiomyocytes of the atria don’t directly stimulate
contraction of the cardiomyocytes of the ventricles?
● Fibrous skeleton cells do not have gap junctions for ion passage
● Chambers are separated by non-contractile adipose tissue
● Purkinje fibers in the fibrous skeleton have inactive sarcomeres
● There are large deposits of glycogen in the fibrous skeleton
Correct Answer: Fibrous skeleton cells do not have gap junctions for ion passage
(Fill in the blank) Identify the cells outlined in black in the image below:

Answer: Purkinje Fibers. Note: This is a PAS-stained slide, the high levels of stored glycogen in
these cells leading to their intense magenta staining.
Purkinje fibers can be best classified as specialized:
● Endothelial cells
● Cardiac muscle cells
● Mesothelial cells
● Fibroblasts
● Autonomic neurons
Correct Answer: Cardiac muscle cells
Which types of blood vessels have the largest lumens?
● Elastic Artery (Aorta)
● Large Vein (Vena Cava)
● Muscular Artery
● Medium Vein
● Arterioles
● Venules
Correct Answer: Large Vein (Vena Cava)
Which types of blood vessels have the thickest walls?
● Elastic Artery (Aorta)
● Large Vein (Vena Cava)
● Muscular Artery
● Medium Vein
● Arterioles
● Venules
Correct Answer: Elastic Artery (Aorta)
What type of tissue is the tunica intima composed of?
● Mesothelium and Loose CT
● Cardiomyocytes and Endomysium
● Endothelium and Loose CT
● Dense Irregular CT
● Smooth Muscle and Collagen Fibers
Correct Answer: Endothelium and Loose CT
What type of tissue is the tunica media composed of?
● Mesothelium and Loose CT
● Cardiomyocytes and Endomysium
● Endothelium and Loose CT
● Dense Irregular CT
● Smooth Muscle and Collagen Fibers
Correct Answer: Smooth Muscle and Collagen Fibers
What type of tissue is the tunica adventitia composed of?
● Mesothelium and Loose CT
● Cardiomyocytes and Endomysium
● Endothelium and Loose CT
● Dense Irregular CT
● Smooth Muscle and Collagen Fibers
Correct Answer: Dense Irregular CT
Which tunic houses the nervi and vasa vasorum?
● Tunica Intima
● Tunica Media
● Tunica Adventitia
Correct Answer: Tunica Adventitia
Which types of blood vessels allow the extravasation and migration of white blood cells?
● Elastic Artery (Aorta)
● Large Vein (Vena Cava)
● Muscular Artery
● Medium Vein
● Arterioles
● Venules
Correct Answer: Venules
(Fill in the blank) Identify the structure pointed by the red arrow in the image below.

Correct Answer: Elastic Lamina


Which of the following tunics is the thickest layer in the wall of elastic arteries?
● Tunica Intima
● Tunica Media
● Tunica Adventitia
Correct Answer: Tunica Media
Which of the following tunics is the thickest layer in the wall of large veins?
● Tunica Intima
● Tunica Media
● Tunica Adventitia
Correct Answer: Tunica Adventitia
Which of the following types of tissue is uniquely found in the tunica adventitia of large veins?
● Endothelium
● Loose CT
● Mesothelium
● Dense CT
● Smooth muscle
Correct Answer: Smooth muscle
Is the artery pictured below classified as an elastic artery?
● Yes
● No
Correct answer: No
Identify the structure indicated by the black arrow:

Correct Answer: Internal Elastic Lamina


Identify the structure indicated by the black arrow:
Correct Answer: External Elastic Lamina
Which of the following are typical histological features of muscular arteries?
● Thin tunica media
● Irregular shaped lumen
● Thick tunica adventitia
● Absence of internal elastic laminae
● Presence of external elastic laminae
Correct Answer: Presence of external elastic laminae
Which of the following are typical histological features of medium veins?
● Thick tunica media
● Irregular shaped lumen
● Thin tunica adventitia
● Presence of internal elastic laminae
● Presence of smooth muscle in tunica adventitia
Correct Answer: Irregular shaped lumen
Identify the blood vessel pictured in the middle of the image below:

● Elastic Artery
● Large Vein
● Muscular Artery
● Medium Vein
● Arterioles
Correct Answer: Medium Vein
Identify the cell indicated by the black arrow in the image below:

Answer: Smooth muscle cell


Identify the cell indicated by the black arrow in the image below:

Answer: Endothelial cell


How many layers of smooth muscle cell do you see in the tunica media of the blood vessel
outlined in black pictured below?
● 1
● 2
● 3
● 4
● 5
Correct answer: 2
How many layers of smooth muscle cell do you see in the tunica media of the blood vessel
pictured below?

● 1
● 2
● 3
● 5
● 8
Correct answer: 5
Identify the blood vessel pictured in the image below?
● Elastic artery
● Medium vein
● Arteriole
● Venule
● Muscular artery
Correct answer: Muscular artery
Identify the blood vessel outlined in black in the image below?

● Elastic artery
● Medium vein
● Arteriole
● Venule
● Muscular artery
Correct answer: Venule
Which type of blood vessel directly drains blood from capillary beds in an arterial portal system?
● Muscular arteries
● Venules
● Medium veins
● Arterioles
Correct answer: Arterioles
Which of the following types of capillaries are primarily found in the brain where capillary
exchange is highly restricted?
● Continuous capillaries
● Fenestrated capillaries
● Discontinuous capillaries
● Sinusoids
Correct answer: Continuous capillaries
Which type of transport is used during capillary exchange in fenestrated capillaries?
● Transcellular transport only
● Paracellular transport only
● Transcellular and paracellular transport
Correct answer: Transcellular and paracellular transport
Which type of capillary is the most permeable?
● Continuous capillaries
● Fenestrated capillaries
● Discontinuous capillaries
Correct answer: Discontinuous capillaries
Sinusoids are also known as which type of vessel?
● Continuous capillaries
● Arterioles
● Fenestrated capillaries
● Post-capillary venules
● Discontinuous capillaries
Correct answer: Discontinuous capillaries
ANATOMY LAB
Match the structures with their labels:

1. Sinoatrial branch of the RCA


2. Right coronary artery (RCA)
3. Right marginal artery
4. Left coronary artery
5. Left anterior descendinf (LAD)
6. Diagonal branch of the LAD
Match the structures with their labels:
1. Right coronary artery
2. Atrio-ventricular branch
3. Right marginal artery
4. Posterior interventricular artery
5. Middle cardiac vein
6. Coronary sinus
Coronary angiography done in advance of coronary bypass surgery on a 54-year-old female
reveals that she has a “right dominant heart.” This condition:
● Indicates that the posterior interventricular artery is derived from the right coronary
artery
● Indicates that the right and left coronary arteries are derived from a common trunk
● Is a major correlate with coronary artery disease
● Is frequently associated with atrial fibrillation
● Is present in <10% of female patients
Correct answer: Indicates that the posterior interventricular artery is derived from the right
coronary artery
During the dissection of the right ventricle of the heart, you see the muscular bridge between the
interventricular septum and the base of the anterior papillary muscle. Your teammates say that it
is the moderator band. Which of the following statements characterizes this structure correctly?
● It prevents the prolapse of the mitral valve
● It carries the right bundle branch to innervate the papillary muscle
● It supplies the internal structures of the heart with oxygen
● It coordinated the contraction of the right atrium
● It has no special function
Correct answer: It carries the right bundle branch to innervate the papillary muscle
OLM #14 - MCB
Drag each of the lipoproteins to their corresponding description.
● Largest, least dense. Contains mostly triglyceride: Chylomicrons
● 2nd largest and 2nd least dense. Carries mostly triglyceride: VLDL
● 2nd smallest and 2nd most dense. Carries mostly cholesterol and cholesterol esters:
LDL
● Smallest, most dense. Carries mostly cholesterol and cholesterol esters: HDL
Drag each of the lipoproteins to the corresponding function.
● Distributes cholesterol to tissues throughout the body: LDL
● Returns cholesterol from tissues throughout the body to the liver: HDL
● Carries dietary triglycerides from the intestinal epithelium to tissues throughout the
body: Chylomicrons
● Carries endogenously synthesized triglycerides from the liver to tissues throughout the
body: VLDL
In the below diagram of a lipoprotein electrophoresis, what is the order of lipoproteins from top
to bottom (alpha, pre-beta, beta, origin). Nb. The anode is on the top and origin is on the bottom.

● Chylomicrons, LDL, VLDL, HDL


● HDL, LDL, VLDL, Chylomicrons
● Chylomicrons, VLDL, LDL, HDL
● HDL, VLDL, LDL, Chylomicrons
Correct answer: HDL, VLDL, LDL, Chylomicrons
Drag each of the following components of the exogenous lipoprotein transport pathway to their
corresponding function.
● Provides ApoE and ApoC-II to nascent chylomicrons in the blood, to form mature
chylomicrons: HDL
● Lipoprotein formed in enterocytes and released into lymph: Nascent chylomicron
● Apoprotein synthesized in the intestinal epithelium needed to initiate chylomicron
synthesis: ApoB-48
● Enzyme on capillary walls, digests chylomicron triglycerides to 2 free fatty acids and
monoacylglycerol: Lipoprotein lipase (LPL)
● Hepatic surface protein that binds ApoE on chylomicron remnants: LDL-Receptor
● Lipoprotein that circulates in blood, depleted of dietary triglycerides, delivers dietary
cholesterol to the liver: Chylomicron remnant
● Packages lipids with ApoB-48 in the endoplasmic reticulum of enterocytes: Microsomal
triglyceride transfer protein (MTP)
● Apoprotein that activates lipoprotein lipase: ApoC-II
● Lipoprotein that circulates in blood distributing dietary triglycerides to peripheral
tissues: Mature chylomicron
Drag each of the following components of the endogenous lipoprotein transport pathway to their
corresponding function.
● Lipoprotein that carries mostly endogenously synthesized cholesterol and cholesterol
esters, delivers cholesterol to peripheral tissues, and returns to the liver by ApoB-100
mediated uptake: LDL
● Provides ApoE and ApoC-II to nascent VLDL in the blood, to form mature VLDL:
HDL
● Lipoprotein initially formed in the liver and released into circulation: Nascent VLDL
● Apoprotein synthesized in the liver needed to initiate VLDL synthesis: ApoB-100
● Enzyme on capillary walls, digests VLDL triglycerides to 2 free fatty acids and
monoacylglycerol: Lipoprotein lipase (LPL)
● Hepatic surface protein that binds ApoE on IDL: LDL-Receptor
● Lipoprotein that circulates in blood, depleted of endogenous triglycerides, 50% are
returned to liver by ApoE-mediated uptake and 50% are metabolized to LDL: IDL
● Hepatic surface protein that binds ApoB-100 on LDL: LDL-Receptor
● Packages lipids with ApoB-100 in the endoplasmic reticulum of hepatocytes:
Microsomal triglyceride transfer protein (MTP)
● Hepatic protease that degrades the LDL-Receptor: PCSK9
● Apoprotein that activates lipoprotein lipase: ApoC-II
● Lipoprotein that circulates in blood distributing endogenous triglycerides to peripheral
tissues: Mature VLDL
Which of the following is true of Lipoprotein(a). (Choose all correct answers).
● A modified HDL particle
● A modified LDL particle
● Coupled to a glycoprotein with Kringle repeats
● Coupled to a fatty acid with Kringle repeats
● Elevated levels decrease risk for cardiovascular disease
● Elevated levels increase risk for cardiovascular disease
Correct answers:
● A modified LDL particle
● Coupled to a glycoprotein with Kringle repeats
● Elevated levels increase risk for cardiovascular disease
By which mechanism do PCSK9 inhibitors decrease plasma cholesterol levels?
● They directly inhibit synthesis of plasma cholesterol
● They inhibit degradation of the LDL-Receptor, which leads to increased clearance of
plasma LDL by hepatic LDL-receptors
● They inhibit synthesis of intracellular hepatic cholesterol which leads to increased
clearance of plasma LDL by the LDL-receptor
● They block bile acid reabsorption, causing hepatic cholesterol to be diverted to the bile,
which leads to increased clearance of plasma LDL by hepatic LDL-receptors
● They limit absorption of dietary cholesterol and reabsorption of biliary cholesterol,
which leads to increased clearance of plasma LDL by hepatic LDL-receptors
● They inhibit lipolysis and release of fatty acids in adipose, thereby inhibiting VLDL
production in the liver and thus lowering LDL levels
Correct answer: They inhibit degradation of the LDL-Receptor, which leads to increased
clearance of plasma LDL by hepatic LDL-receptors
By which mechanism do enterocytes produce the smaller ApoB-48, compared to the full-sized
ApoB-100 produced in hepatocytes?
● Expression of alternative genes
● Alternative splicing of pre-mRNA
● RNA editing
● Proteolytic processing of the larger protein by an endoproteases
● Proteolytic processing of the larger protein by the proteosome
Correct answer: RNA editing
Drag each of the following components of the reverse cholesterol transport pathway to their
corresponding function.
● Transport protein that exchanges triglyceride from VLDL with cholesterol esters from
HDL: CETP
● Enzyme carried by HDL particles that converts surface cholesterol to cholesterol esters
which move to the core: LCAT
● A smaller form of spherical HDL that forms when cholesterol in discoidal HDL is
converted to cholesterol esters: HDL3
● Apoprotein produced by the liver and intestines needed for initiation on HDL synthesis:
ApoA-I
● A larger form of spherical HDL that swells as it incorporates more cholesterol esters and
triglycerides from VLDL: HDL2
● A hepatocyte scavenger receptor that mediates direct uptake of HDL cholesterol: SR-B1
● The initial form of HDL consisting of phospholipid and free cholesterol but not
cholesterol esters: Discoidal HDL
● Transfers phospholipid and cholesterol to ApoA-1 and discoidal HDL: ABCA1
Nicotinic acid has been used for treatment of hypercholesterolemia, but its clinical utility is
being debated. By which mechanism does nicotinic acid reduce plasma cholesterol?
● It directly inhibits synthesis of plasma cholesterol
● It inhibits synthesis of intracellular hepatic cholesterol which leads to increased
clearance of plasma LDL by the LDL-receptor
● It blocks bile acid reabsorption, causing hepatic cholesterol to be diverted to the bile,
which leads to increased clearance of plasma LDL by hepatic LDL-receptors
● It limits absorption of dietary cholesterol and reabsorption of biliary cholesterol, which
leads to increased clearance of plasma LDL by hepatic LDL-receptors
● It inhibits lipolysis and release of fatty acids in adipose, thereby inhibiting VLDL
production in the liver and thus lowering LDL levels
Correct answer: It inhibits lipolysis and release of fatty acids in adipose, thereby inhibiting
VLDL production in the liver and thus lowering LDL levels
ANATOMY LAB
Imaging of the thorax in an 18-month-old female reveals a diffuse structure in the superior
mediastinum, representing the thymus gland. All of the following also would be found in the
superior mediastinum except:
● Ascending aorta
● Left brachiocephalic vein
● Vagus nerve
● Brachiocephalic artery
● Phrenic nerve
Correct answer: Ascending aorta
Which of the following structures may be affected by a tumor in the superior mediastinum?
● Arch of the aorta
● Superior vena cava
● Brachiocepalic vein
● Sympathetic trunk
● All of the above
Correct answer: All of the above
Identify the right subclavian artery in this cadaveric image of the superior mediastinum.

Answer – 2
The most likely structure to be compressed by a tumor found above the root of the left lung is
which of the following?
● Arch of the aorta
● Left pulmonary vein
● Azygos vein
● Pulmonary trunk
● Ascending aorta
Correct answer: Arch of the aorta
OLM #16 - MCB
Drag the elevated lipoproteins and lipids to the appropriate hyperlipidemia.
Type I
● Chylomicrons
● Triglyceride
Type IIa
● LDL
● Cholesterol
Type IIb
● LDL
● VLDL
● Cholesterol
● Triglyceride
Type III
● IDL
● Triglyceride
● Cholesterol
Type IV
● VLDL
● Triglyceride
Type V
● Chylomicrons
● VLDL
● Triglyceride
For each of the diagrams of lipoprotein electrophoresis shown below, indicate which
hyperlipidemia is occurring. Drag the letter to the corresponding type. Nb. The anode is on the
top and the origin is on the bottom.

A. Type IIa
B. Type IV
C. Type I
D. Type V
E. Type IIb
F. Type III
Drag the Frederickson classification for each of the hyperlipidemias to the corresponding
common name of the disease.
● Dysbetalipoproteinemia: Type III
● Combined Hyperlipoproteinemia: Type IIb
● Hyperchylomicronemia: Type I
● Familial Hypertriglyceridemia: Type IV
● Familial Hypercholesterolemia: Type IIa
● Combined Hyperchylomicronemia: Type V
Drag the appropriate hyperlipidemia to the corresponding genetic defect.
● Lipoprotein lipase loss-of-function (with a high carbohydrate diet): Type V
● LDL receptor loss-of-function: Type IIa
● ApoB-100 gain-of-function with LDL Receptor loss-of-function: Type IIb
● Lipoprotein lipase loss-of-function (with a low carbohydrate diet): Type I
● ApoB-100 loss-of-function: Type IIa
● ApoE (e2 allele): Type III
● PCSK9 gain-of-function: Type IIa
● ApoC-II loss-of-function (with a low carbohydrate diet): Type I
● ApoA-V loss-of-function: Type V
Drag the clinical presentations found in the various hyperlipidemias to the corresponding type.
Type I
● Eruptive Xanthoma
● Acute Pancreatitis
● Lipemia Retinalis
Type IIa
● Tendonous Xanthoma
● Xanthelasma
● Corneal Arcus
Type IIb
● Tendonous Xanthoma
● Xanthelasma
● Corneal Arcus
Type III
● Palmar Xanthoma
● Tuberoeruptive Xanthoma
Type IV
● Variable Xanthomas
● Acute Pancreatitis
Type V
● Variable Xanthomas
● Acute Pancreatitis
● Lipemia Retinalis
Which of the hyperlipidemias are associated with an increased risk of cardiovascular disease?
(Choose all correct answers)
● Type I
● Type IIa
● Type IIb
● Type III
● Type IV
● Type V
Correct answers:
● Type IIa
● Type IIb
● Type III
● Type IV
● Type V
Which of the following are likely to be beneficial treatment options for homozygous familial
hypercholesterolemia? (Choose all correct answers)
● Statins
● Cholesterol uptake inhibitors
● Liver transplantation
● Mipomersen
● LDL apheresis
● Bile Acid sequestrants
● Lomitapide
Correct answers:
● Liver transplantation
● Mipomersen
● LDL apheresis
● Lomitapide
Which of the following are true regarding abetalipoproteinemia? (Choose all correct answers)
● Autosomal recessive inheritance
● Autosomal dominant inheritance
● Homozygosity for the ApoE (e2) allele
● Deficiency of microsomal triglyceride transfer protein (MTP)
● Deficiency of ABCA1
● HDL is very low
● Chylomicrons and chylomicron remnants are very low
● VLDL, and LDL are very low
● Triglycerides and cholesterol are elevated
● Triglycerides and cholesterol are decreased
Correct answers:
● Autosomal recessive inheritance
● Deficiency of microsomal triglyceride transfer protein (MTP)
● Chylomicrons and chylomicron remnants are very low
● VLDL, and LDL are very low
● Triglycerides and cholesterol are decreased
Which of the following is true regarding LCAT deficiency? (Choose all correct answers)
● HDL is very low
● Chylomicrons and chylomicron remnants are very low
● VLDL, and LDL are very low
● Cholesterol accumulates in peripheral tissues
Correct answers:
● HDL is very low
● Cholesterol accumulates in peripheral tissues
Which of the following is true regarding Tangier disease? (Choose all correct answers)
● HDL is very low
● Chylomicrons and chylomicron remnants are very low
● VLDL, and LDL are very low
● Cholesterol accumulates in peripheral tissues
● Deficiency of microsomal triglyceride transfer protein (MTP)
● Deficiency of ABCA1
Correct answers:
● HDL is very low
● Cholesterol accumulates in peripheral tissues
● Deficiency of ABCA1
Drag the various clinical presentations found in the hypolipidemias to the corresponding disease.
Abetalipoproteinemia
● Fatty liver
● Steatorrhea
● Failure to thrive
● Secondary deficiencies of fat-soluble vitamins
LCAT Deficiency
● Corneal opacification (Fish-eye disease)
● Hemolytic anemia
● Kidney disease
Tangier Disease
● Orange tonsils
● Hepatosplenomegaly
● Accumulation of foam cells
Which of the hypolipidemias are associated with an increased risk of cardiovascular disease?
(Choose all correct answers)
● Abetalipoproteinemia
● LCAT deficiency
● Tangier disease
Correct answers: LCAT deficiency, Tangier disease
OLM #17 - Anatomy
A 65-year-old man with a history of coronary artery disease comes to the physician complaining
of progressive exertional shortness of breath, fatigue, and lower extremity swelling. His medical
history is significant for a myocardial infarction 5 years ago and an electronic pacemaker
implanted 2 years ago. His chest x-ray is shown below. A segment of one of the leads is
highlighted (arrow). The highlighted segment most likely lies within which of the following
structures?

● Right ventricle
● Right atrium
● Pulmonary artery
● Coronary groove
● Anterior interventricular sulcus
Correct answer: Coronary groove
A 45-year-old man comes to the emergency department with severe dyspnea (shortness of
breath) and chest pain on inspiration. He recently returned from a transatlantic trip. Chest X-ray
reveals a pulmonary embolism (blot clot) and he is diagnosed with respiratory failure. In the
x-ray film shown below, which of the following structures of the heart is pointed by the black
arrows?
● Arch of aorta
● Left ventricle
● Right ventricle
● Right atrium
● Left atrium
Correct answer: Right atrium
Identify the left crus of the diaphragm on this CT image:

Correct Answer: 2
Choose the appropriate lettered site or structure in this CT scan of the thorax from a 42-year-old
man who complains of chest pain and breathing problems. His electrocardiogram shows left
ventricular hypertrophy. Stenosis of which structure may produce left ventricular hypertrophy?

Correct Answer: C
Choose the appropriate lettered site or structure in this CT scan of the thorax. Which structure
lies on the right side of the aortic arch and ascending aorta?

Correct Answer: A
ANATOMY LAB
Dissecting the posterior mediastinum, you revealed some nerve plexus on the posterior surface of
the esophagus. This plexus consists of nerve fibers of which of the following nerves?
Left vagus Right phrenic Right vagus Left phrenic Splanchnic
Correct answer: Right vagus
A 44-year-old patient hospitalized a week ago with the retropharyngeal abscess undergoes
computerized tomography of his neck and thorax. The temperature is 38.2, pulse 92/min. He was
admitted in hard condition and antibiotic therapy does not seem to be effective. The radiologist
reveals a collection of fluids behind the esophagus of the patient. This finding suggest that
infection has spread to which of the following spaces?
Pretracheal space Suprasternal space Paravertebral space Prevertebral (danger space) Visceral
space
Correct answer: Prevertebral (danger space)
Dissecting the posterior mediastinum, you revealed a thin-walled vessel behind the esophagus
passing between the azygos vein on the right and the descending aorta on the left. What is that
vessel?
Intercostal vein Pericardiacophrenic artery Intercostal artery Thoracic duct Ascending aorta
Correct answer: Thoracic duct
In the drawing below, of the 5 openings of the thoracic diaphragm lettered A, B, C, D, & E.
Which one transmits the azygos vein?

Correct answer: D
OLM #18 - Physiology
What happens at the end of ventricular isovolumic relaxation?
● The A-V valves closes
● The aortic valve opens
● The aortic valve closes
● The mitral valve opens
● The pulmonary valve closes
Answer: The mitral valve opens. At the end of isovolumic relaxation, the mitral and tricuspid
valves open, which is followed by the period of diastolic filling.
Which event is associated with the first heart sound?
● Closing of the aortic valve
● Inrushing of blood into the ventricles during diastole
● Beginning of diastole
● Opening of the A-V valves
● Closing of the A-V valves
Answer: Closing of the A-V valves. The first heart sound by definition occurs just after the
ventricular pressure exceeds the atrial pressure, which causes the A-V valves to mechanically
close. The second heart sound occurs when the aortic and pulmonary valves close.
What normally causes the cardiac output curve to shift to the left along the right atrial pressure
axis?
● Surgically opening the chest
● Severe cardiac tamponade
● Breathing against a negative pressure
● Playing a trumpet
● Positive-pressure breathing
Answer: Breathing against a negative pressure. Several factors can cause the cardiac output to
shift to the right or to the left. Among those are surgically opening the chest, which makes the
cardiac output curve shift 4 mm Hg to the right, and severe cardiac tamponade, which increases
the pressure inside the pericardium, thus tending to collapse the heart, particularly the atria.
Playing a trumpet or positive pressure breathing tremendously increases the intrapleural pressure,
thus collapsing the atria and shifting the cardiac output curve to the right. Breathing against a
negative pressure shifts the cardiac output curve to the left.
What will elevate the plateau of the cardiac output curve?
● Surgically opening the thoracic cage
● Connecting a patient to a mechanical ventilator
● Cardiac tamponade
● Increasing parasympathetic stimulation of the heart
● Increasing sympathetic stimulation of the heart
Answer: Increasing sympathetic stimulation of the heart. The plateau level of the cardiac output
curve, which is one measure of cardiac contractility, decreases in several circumstances. Some of
these circumstances include severe cardiac tamponade, which increases the pressure in the
pericardial space, and increasing parasympathetic stimulation of the heart. Increased sympathetic
stimulation of the heart increases the level of the cardiac output curve by increasing heart rate
and contractility.
What is normally associated with an increased cardiac output?
● Increased parasympathetic stimulation
● Atrioventricular (A-V) fistula
● Decreased blood volume
● Polycythemia
● Severe aortic regurgitation
Answer: Atrioventricular (A-V) fistula. Cardiac output increases in several conditions because of
increased venous return. A-V fistulae also cause a decreased resistance to venous return, thus
increasing cardiac output. Cardiac output decreases in patients with hypovolemia, severe aortic
regurgitation, and polycythemia. The hematocrit level is high in polycythemia, which increases
resistance to venous return.
Which condition would be expected to decrease mean systemic filling pressure?
● Norepinephrine administration
● Increased blood volume
● Increased sympathetic stimulation
● Increased venous compliance
● Skeletal muscle contraction
Answer: Increased venous compliance. Mean systemic filling pressure is a measure of the
tightness of fit of the blood in the circulation. Mean systemic filling pressure is increased by
factors that increase blood volume and decrease the vascular compliance. Therefore, decreased
venous compliance, not increased compliance, would cause an increase in mean systemic filling
pressure. Norepinephrine administration and sympathetic stimulation cause arteriolar
vasoconstriction and decreased vascular compliance, resulting in an increase in mean systemic
filling pressure. Increased blood volume and skeletal muscle contraction, which cause a
contraction of the vasculature, also increase this filling pressure.
Which of the following would decrease cardiac output?
● Increased stroke volume
● Increased heart rate
● Increased mean systemic filling pressure
● Increased resistance to venous return
● Increased venous return
Answer: Increased resistance to venous return. Venous return (or cardiac output) is equal to the
mean systemic filling pressure minus the right atrial pressure divided by resistance to venous
return. Thus, an increase in resistance to venous return decreases venous return and cardiac
output.
If a patient has an oxygen consumption of 240 ml/min, a pulmonary vein oxygen concentration
of 180 ml/l of blood, and a pulmonary artery oxygen concentration of 160 ml/l of blood units,
what is the cardiac output in l/min?
● 8
● 10
● 12
● 16
● 20
Answer: 12. This problem concerns the Fick principle for determining cardiac output. The
formula for cardiac output is oxygen absorbed per minute by the lungs divided by the
arterial-venous oxygen difference. In this problem, oxygen consumption of the body is 240
ml/min, and in a steady-state condition, this would exactly equal the oxygen absorbed by the
lungs. Therefore, by inserting these values into the equation, we see that the cardiac output will
equal 12 l/min.
A 35-year-old man has been diagnosed with a vitamin B1 deficiency. Oxygen consumption in
this man is 400 ml/min. In addition, pulmonary vein oxygen concentration is 200 ml/l of blood,
and pulmonary artery oxygen concentration is 150 ml/l of blood. What is the cardiac output
(l/min) in this man?
● A. 4.0
● B. 5.0
● C. 6.0
● D. 7.0
● E. 8.0
Answer: 8.0. According to the Fick principle,
Cardiac output = Oxygen absorbed by lungs(ml/min) (400) divided by
Arterio-venous oxygen difference (200 – 150 ml/l)
Cardiac output = 400/50 or 8 l/min
OLM #19 - Pathology
Where are the vasa vasorum:
● Intima
● Media
● Adventitia
● Smooth muscles
Correct answer: Adventitia
What is the characteristic lesion of Atherosclerosis:
● Fatty streak
● Atheroma
● Foam cells
● Intracellular and extracellular lipids
Correct answer: Atheroma
What are foam cells:
● Macrophages
● Lymphocytes
● Platelets
● RBCs
● Neutrophils
Correct answer: Macrophages
Which of the following is a major modifiable risk factor for AS:
● Age
● Male gender
● Family history
● Obesity
● Hypertension
Correct answer: Hypertension
Which of the following is a major non-modifiable risk factor for AS:
● Hyperlipidemia
● Hypertension
● Cigarette smoking
● Diabetes mellitus
● Male gender
Correct answer: Male gender
What is the clinical consequence of Monckeberg Medial Calcific Sclerosis:
● Aneurysm formation
● Ischemic heart disease
● Cerebral infarction (stroke)
● Gangrene of lower extremities
● No clinical consequence
Correct answer: No clinical consequence
OLM #20 - Pathology
Raynaud’s phenomenon reflects:
● Dilation of local small arteries
● Spasm of local small arteries
● Spasm of local small veins
● Dilation of local small veins
Correct answer: Spasm of local small arteries
What is the most common cause for secondary hypertension:
● Psychogenic
● Renovascular
● Adrenocortical hyperfunction
● Hyperthyroidism
● Sleep apnea
Correct answer: Renovascular
What is the most common cause for renovascular hypertension:
● Acute glomerulonephritis
● Renal artery stenosis
● Renin-producing tumors
● Renal vasculitis
● Polycystic kidney disease
Correct answer: Renal artery stenosis
Fibromuscular Hyperplasia radiologically on X-Ray appears as:
● A string of beads
● A corkscrew
● A Holly Leaf
● A ring around an artery
Correct answer: A string of beads
What is the most common complication of hypertension:
● Concentric left ventricular hypertrophy
● Stroke
● Hypertensive retinopathy
● Acute MI
● Hyaline arteriolosclerosis
Correct answer: Concentric left ventricular hypertrophy
OLM: Pathology - Aneurysms

What is the most common type of aneurysms:

● Abdominal Aortic Aneurysms


● Syphilitic Aneurysm
● Berry Aneurysm
● Mycotic aneurysm

Correct answer: Abdominal Aortic Aneurysms

What is the most common cause for aneurysms:

● Congenital defects
● Syphilis
● Atherosclerosis
● Vasculitis
● Syphilis

Correct answer: Atherosclerosis

Berry aneurysm is associated with which of the following genetic conditions:

● Marfan syndrome
● Ehlers-Danlos syndrome
● Adult polycystic kidney disease
● Down syndrome
● Hemochromatosis

Correct answer: Adult polycystic kidney disease

What is the most common cause of death in aortic dissection:

● Cardiac tamponade
● Stroke
● MI
● Subarachnoid hemorrhage

Correct answer: Cardiac tamponade

What is the most likely cause of aortic dissection in a 60-year-old male:

● Marfan syndrome
● Ehlers Danlos syndrome
● Hypertension
● Adult polycystic kidney disease
● Syphilis

Correct answer: Hypertension

OLM: Pathology - Shock

What is the most common cause for hypovolemic shock:

● Hemorrhage
● Diarrhea
● Severe burns
● Excessive vomiting
● Excessive sweating

Correct answer: Hemorrhage

What is the most common cause for cardiogenic shock:

● Myocardial infarction
● Arrhythmia
● Cardiac tamponade
● Pulmonary embolism
● DVT

Correct answer: Myocardial infarction

Anaphylactic shock is characterized by which type of hypersensitivity:

● Type 1
● Type 2
● Type 3
● Type 4

Correct answer: Type 1

What is the most common type of shocks:

● Anaphylactic shock
● Septic shock
● Hypovolemic shock
● Cardiogenic shock
● Neurogenic shock
Correct answer: Septic shock

Lecture: Pharmacology - Drugs for Hypertension

A 64-year-old Caucasian woman presents to the clinic for her annual physical check-up. She has
recently lost her husband and she explains that her budget is limited as she has no healthcare
coverage. She has no other complaints. Her physical exam is unremarkable. The average of 2
blood pressure readings in her right arm is 156/88 mm Hg. Her physical exam is unremarkable.
She has no history of myocardial infarction, stroke, kidney disease, or heart failure. You
recommend that she measures her BP at home and to return six weeks later for further
evaluation. The average BP from multiple clinic and home readings is 160/86 mm Hg. Her lipid
profile is normal. You wish to prescribe her a drug to manage the blood pressure. Which of the
following medications would be the optimal first line agent in her case for her hypertension?

● Amlodipine
● Captopril
● Hydrochlorothiazide
● Metoprolol
● Nitrendipine

Correct answer: Hydrochlorothiazide

A 61-year-old African-American male non-smoker with metabolic syndrome. His physical exam
is unremarkable except for obesity. He has no history of myocardial infarction, stroke, kidney
disease, or heart failure. After several weeks of trying therapeutic lifestyle changes, he returns
and upon examination his parameters (BP, weight etc.) have not changed significantly. Which of
the following combinations of medications would be optimal to initiate treatment for his
hypertension to achieve target blood pressure in this patient?

● Amlodipine + prazosin
● Atenolol + lisinopril
● Indapamide + Hydrochlorothiazide
● Hydrochlorothiazide + lisinopril
● Nitrendipine + propranolol

Correct answer: Hydrochlorothiazide + lisinopril

A 63-year-old Caucasian man with type 2 diabetes has developed hypertension. His blood
pressure measurements average 151/92 mm Hg over the span of several weeks of measuring at
home and in the clinic. He also has albuminuria. Which of the following agents would be the
most appropriate first line agent or the treatment of his elevated blood pressure?

● Bisoprolol
● Indapamide
● Nifedipine
● Telmisartan
● Prazosin

Correct answer: Telmisartan

A 60-year-old man reported to his physician that for the past 2 days he felt very dizzy and faint
when he stood up rapidly. The man had a history of essential hypertension that was poorly
controlled with losartan and hydrochlorothiazide. One week earlier, he was diagnosed with
prostatic hyperplasia, and the physician added prazosin to the therapy. Which of the following is
the most likely mechanism of action of prazosin in hypertension?

● Blockade of vascular alpha-1 receptors


● Increased urinary excretion of sodium
● Opening of potassium channels
● Blockade of L-type calcium channels
● Blockade of myocardial beta-1 receptors

Correct answer: Blockade of vascular alpha-1 receptors

A 67-year-old man was found to have a plasma calcium level of 12.2 mg/dL during a follow-up
visit. The man had a 3-year history of Hodgkin lymphoma. He was recently diagnosed with
nephrolithiasis for which he had been treated with hydrochlorothiazide for the past 3 weeks.
Which of the following best explains the most likely mechanism of thiazide-induced
hypercalcemia?

● Activation of the Na+/Ca2+ exchanger in the distal tubule


● Increased Ca2+ reabsorption in the proximal tubule
● Decreased secretion of parathyroid hormone
● Decreased renal excretion of vitamin D
● Activation of Na+/K+/2Cl- symporter in the thick ascending loop of Henle

Correct answer: Activation of the Na+/Ca2+ exchanger in the distal tubule

OLM: Pathology - Venous & Lymphatic System Disorders & Tumors

What is the most common site for varicose veins:

● Superficial saphenous veins


● Distal esophagus
● Anorectal region
● Left scrotal sac
Correct answer: Superficial saphenous veins

What is the most common cause for superior vena cava syndrome:

● Bronchogenic carcinoma
● Renal cell carcinoma
● Hepatocellular carcinoma
● Hepatic angiosarcoma
● DVT

Correct answer: Bronchogenic carcinoma

Bacillary angiomatosis lesions look like lesions of:

● Pyogenic granuloma
● Kaposi sarcoma
● Hepatic angiosarcoma
● Capillary hemangiomas
● Cavernous hemangiomas

Correct answer: Kaposi sarcoma

Kaposi sarcoma is the most common cancer in:

● Smokers
● AIDS patients
● Coal miners
● Turner syndrome
● Von-Hippel-Lindau disease

Correct answer: AIDS patients

Angiosarcoma is positive for the endothelial cell marker:

● CD31
● CD14
● CD4
● CD3
● CD30

Correct answer: CD31

Lecture: Pharmacology - Drugs for the Management of Dyslipidemias


A 63-year-old obese woman was found to have a total cholesterol level of 360 mg/dL (normal <
200 mg/dL), despite many months of lovastatin treatment. The physician decided to add
ezetimibe to the therapeutic regimen. Which of the following cells represents the main site of
action of ezetimibe?

● Adipocytes
● Capillary endothelial cells
● Platelets
● Hepatocytes
● Intestinal epithelial cells

Correct answer: Intestinal epithelial cells

A 23-year-old woman suffering from familial hypertriglyceridemia was screened with a blood
lipid profile during a routine visit. Relevant laboratory test results were uric acid 15 mg/dL
(normal 4.0 - 8.5 mg/dL), total cholesterol 170 mg/dL (normal < 200 mg/dL), triglycerides 1230
mg/dL (normal < 200 mg/dL). A lipid-lowering drug was prescribed. Which of the following
was most likely the mechanism of action of that was prescribed?

● Increase of lipid synthesis by adipose tissue


● Downregulation of low-density lipoprotein (LDL) receptors in the liver
● Inhibition HMG CoA reductase activity in the liver
● Stimulation of lipoprotein lipase synthesis
● Decreased absorption of exogenous cholesterol

Correct answer: Stimulation of lipoprotein lipase synthesis

A 55-year-old woman was brought to the hospital because of sudden unilateral blindness and
inability to move the extremities of the contralateral body side. All symptoms disappeared a half
hour later. An angiogram revealed a 55% stenosis of the right carotid artery. An appropriate
therapy was prescribed. Which of the following drugs given chronically would be most likely to
contribute to a decreased risk of further stroke in this patient?

● Isosorbide mononitrate
● Furosemide
● Simastatin
● Esmolol
● Gemfibrozil

Correct answer: Simastatin

During a routine follow-up visit, a 57-year-old man was found to have the following lab results
alanine aminotransferase 120 UL (normal 8 – 20UL), aspartate aminotransferase 108 UL
(normal 8 – 20UL). The man had been discharged from the hospital after an acute myocardial
infarction 6 weeks earlier and was on an appropriate post-discharge therapy. Which of the
following drugs most likely caused the lab results?

● Atenolol
● Clopidogrel
● Captopril
● Lovastatin
● Warfarin

Correct answer: Lovastatin

OLM: Pathology - Vasculitis

What is the most common symptom of giant cell arteritis:

● Unilateral headache
● Visual disturbances
● Occasional blindness
● Jaw claudication
● Polymyalgia rheumatica

Correct answer: Unilateral headache

Large vessel vasculitis is characterized by which type of hypersensitivity:

● Type 1
● Type 2
● Type 3
● Type 4

Correct answer: Type 4

Buerger’s disease is highly assosiated with:

● Smoking
● Hypertension
● Diabetes
● Alcoholism
● Liver disease

Correct answer: Smoking

Polyarteritis Nodosa spares which organ:

● Heart
● Lung
● Liver
● GIT
● Kidney

Correct answer: Lung

Granulomatosis with polyangiitis is highly associated with:

● PR3-ANCA
● MPO-ANCA
● ANA
● RF
● ds-DNA

Correct answer: PR3-ANCA

Microscopic Polyangiitis is highly associated with:

● PR3-ANCA
● MPO-ANCA
● ANA
● RF
● ds-DNA

Correct answer: MPO-ANCA

What is the most common vasculitis in children:

● Henoch-Schönlein Purpura
● Bechet’s Disease
● Kawasaki’s Disease
● Buerger’s disease
● Polyarteritis Nodosa

Correct answer: Henoch-Schönlein Purpura

OLM: Pathology - Embolism

What is the most common type of embolisms:

● Thromboembolism
● Fat embolism
● Bone marrow embolism
● Air embolism
● Amniotic fluid embolism

Correct answer: Thromboembolism

What is the most serious complication of DVT:

● Myocardial infarction
● Stasis Dermatitis
● Superficial vein varicosities
● Pulmonary embolism
● Stroke

Correct answer: Pulmonary embolism

What is the most common site of systemic arterial emboli:

● Lower extremities
● Brain
● Small Intestine
● Kidney
● Liver

Correct answer: Lower extremities

Which of the following organs can have a red infarction:

● Heart
● Kidney
● Spleen
● Lung

Correct answer: Lung

Lecture: Pharmacology - Drugs for the Management of Heart Failure

Name a drug can increase the synthesis of cyclic adenosine monophosphate(cAMP) in the heart.

Correct Answer: Dobutamine

Chronic use of this mineralocorticoid receptor blocker has been shown to reduce mortality in
patients with severe heart failure.

Correct Answer: Spironolactone


A 64-year-old man recently diagnosed with stage C heart failure was admitted to the hospital for
evaluation. His left ventricular ejection fraction was determined to be 30% at rest. Treatment that
included digoxin was started. Which of the following cardiovascular parameters did digoxin
most likely increase in this patient?

● Stroke volume
● Total peripheral resistance
● Oxygen consumption of the heart
● End-diastolic volume
● Heart rate

Correct answer: Stroke volume

A 73-year-old man was recently diagnosed with severe heart failure, and the physician started
treatment with propranolol, captopril, and digoxin. Diuretic therapy was also included. Which of
the following pairs of diuretics would have been most appropriate for this patient?

● Furosemide and mannitol


● Furosemide and spironolactone
● Hydrochlorothiazide and acetazolamide
● Triamterene and acetazolamide
● Hydrochlorothiazide and mannitol

Correct answer: Furosemide and spironolactone

A 52-year-old alcoholic man suffering from liver cirrhosis was admitted to the emergency
department because of a 2-week history of nausea, vomiting, and lower abdominal cramps.
Physical examination showed a tense abdomen with prominent veins, and 3+ ascites was noted
by shifting dullness and a fluid wave. A diagnosis of ascites was made, and an appropriate
diuretic therapy was started. Which of the following diuretics was most likely prescribed for this
patient?

● Acetazolamide
● Ethacrynic acid
● Prazosin
● Triamterene
● Spironolactone

Correct answer: Ethacrynic acid

A 54-year-old woman presented to the clinic complaining of palpitations. One month earlier, the
woman was diagnosed with stage C heart failure and started treatment with metoprolol, digoxin,
and captopril. Her medications also included estrogen and a calcium supplement for
postmenopausal osteoporosis. The patient’s vital signs were blood pressure 145/90, pulse 130
bpm. An electrocardiogram showed ventricular tachycardia. Significant plasma levels on
admission were K+ 5.8 mEq/L, Ca2+ 12.2 mEq/L, creatinine 3.5 mg/dL. Which of the following
events most likely triggered the patient’s arrhythmia?

● Decreased cardiac contractility


● Increased serum K+ level
● Captopril-induced vasodilation
● Increased serum Ca2+ level
● Estrogen-induced hypertension

Correct answer: Increased serum K+ level

OLM: Pathology - Hypertrophy, Hypertensive Heart Disease, & Heart Failure

Both atrioventricular (AV) and sinoatrial node (SA) are supplied by:

● Aorta
● Pulmonary artery
● Left anterior descending coronary artery (LAD)
● Left circumflex coronary artery (LCX)
● Right coronary artery (RCA)

Correct answer: Right coronary artery (RCA)

What is the most common cause of concentric LV hypertrophy:

● Pulmonary hypertension
● Pulmonary artery stenosis
● Essential hypertension
● Aortic stenosis
● Mitral stenosis

Correct answer: Essential hypertension

Heart failure cells are:

● Macrophages
● Myocardial cells
● Neutrophils
● Platelets
● Pneumocytes

Correct answer: Macrophages


What is the first cardiac finding in LHF:

● S4 heart sound
● S3 Heart sound
● Mitral valve regurgitation
● Paroxysmal nocturnal dyspnea
● Pulmonary edema

Correct answer: S3 Heart sound

What is the most common cause of AV fistula:

● Surgical shunt for dialysis


● Paget's disease of bone
● Trauma from knife wound
● Thiamine deficiency
● Hyperthyroidism

Correct answer: Trauma from knife wound

Lecture: Pharmacology - Antithrombotic Drugs

A 54-year-old man reported to his physician that in the morning he noticed his urine was cloudy
and red. He had been taking an oral anticoagulant for 3 weeks to treat a deep venous thrombosis
(DVT). If the patient’s symptoms were caused by the anticoagulant, which of the following
adverse drug reactions was most likely involved?

● Overdose toxicity
● Autoimmune reaction
● Idiosyncratic reaction
● Pseudo-allergic reaction
● Immediate allergic reaction

Correct answer: Overdose toxicity

A 61-year-old Asian man with atrial fibrillation presented to his physician complaining of red
urine. He had been receiving a standard dose of warfarin, which is an anticoagulant metabolized
by the CYP2C9 isozyme of cytochrome P450. Which of the following was the most likely cause
of the patient’s disorder?

● Increased protein binding of warfarin


● Decreased renal excretion of warfarin
● Genetic polymorphism of CYP2C9
● Decreased metabolism of CYP2C9
● Increased CYP2C9 synthesis in a person of Asian origin

Correct answer: Decreased metabolism of CYP2C9

A 67-year-old woman presented to the clinic because of progressive swelling and soreness of the
left calf. After physical examination and lab exams, a diagnosis of deep venous thrombosis was
made, and a treatment with enoxaparin was started. Which of the following is the main
advantage of this drug over the standard unfractionated heparin?

● Complete absence of bleeding complications


● Good oral bioavailability
● Lower incidence of drug-induced thrombocytopenia
● Pronounced antiplatelet activity
● Inhibitions of a larger number of coagulation factors

Correct answer: Lower incidence of drug-induced thrombocytopenia

A 69-year-old obese man presented to the clinic because of left calf swelling and pain of 1-day
duration. He stated that a few days before the onset of symptoms, he had arrived home after a
10-hour flight from India. Duplex scanning was performed and revealed clot formation in the
patient’s left calf. The patient did not exhibit signs of pulmonary embolism. Which of the
following pairs of drugs were most likely prescribed to the patient?

● Heparin and abciximab


● Warfarin and abciximab
● Heparin and warfarin
● Alteplase and warfarin
● Alteplase and heparin

Correct answer: Heparin and warfarin

A 54-year-old woman suffering from asthma was brought to the emergency department because
of a sudden onset of left side paralysis. Imaging studies confirmed the diagnosis of
thromboembolic stroke, and the patient started a treatment that included a drug that acts by
blocking platelet adenosine diphosphate (ADP) receptors. Which of the following drugs has this
mechanism of action?

● Warfarin
● Aminocaproic acid
● Alteplase
● Clopidogrel
● Heparin
Correct answer: Clopidogrel

OLM: Physiology - ECG - Excitation of the Heart

In which phase of the ventricular muscle action potential is the sodium permeability the highest?

● Phase 0
● Phase 1
● Phase 2
● Phase 3
● Phase 4

Answer: Phase 0. Sodium permeability is highest during phase 0. Calcium permeability is


highest during phase 2, and potassium is most permeable in phase 3.

Which of the following structures will have the slowest rate of conduction of the cardiac action
potential?

● Atrial muscle
● Anterior internodal pathway
● A-V bundle fibers
● Purkinje fibers
● Ventricular muscle

Answer: A-V bundle fibers. The atrial and ventricular muscles have a relatively rapid rate of
conduction of the cardiac action potential, and the anterior internodal pathway also has fairly
rapid conduction of the impulse. However, the A-V bundle myofibrils have a slow rate of
conduction because their sizes are considerably smaller than the sizes of the normal atrial and
ventricular muscle. In addition, their slow conduction is partly caused by diminished numbers of
gap junctions between successive muscle cells in the conducting pathway, causing a great
resistance to conduction of the excitatory ions from one cell to the next.

If the S-A node discharges at 0.00 seconds, when will the action potential normally arrive at the
epicardial surface at the base of the left ventricle?

● 0.22 second
● 0.18 second
● 0.16 second
● 0.12 second
● 0.09 second

Answer: 0.22 second. After the S-A node discharges, the action potential travels through the
atria, through the A-V bundle system, and finally to the ventricular septum and throughout the
ventricle. The last place that the impulse arrives is at the epicardial surface at the base of the left
ventricle, which requires a transit time of 0.22 second.

Which condition at the A-V node will cause a decrease in heart rate?

● Increased sodium permeability


● Decreased acetylcholine levels
● Increased norepinephrine levels
● Increased potassium permeability
● Increased calcium permeability

Answer: Increased potassium permeability. The increase in potassium permeability causes a


hyperpolarization of the A-V node, which will decrease the heart rate. Increases in sodium
permeability actually partially depolarizes the A-V node, and an increase in norepinephrine
levels increases the heart rate.

If the Purkinje fibers, situated distal to the A-V junction, become the pacemaker of the heart,
what is the expected heart rate?

● 30/min
● 50/min
● 60/min
● 70/min
● 80/min

Answer: 30/min. If the Purkinje fibers are the pacemaker of the heart, the heart rate ranges
between 15 and 40 beats/min. In contrast, the rate of firing of the A-V nodal fibers are 40 to 60
times a minute, and the sinus node fires at 70 to 80 times/min. If the sinus node is blocked for
some reason, the A-V node will take over as the pacemaker, and if the A-V node is blocked, the
Purkinje fibers will take over as the pacemaker of the heart.

A patient had an ECG at the local emergency department. The attending physician stated that the
patient had an A-V nodal rhythm. What is the likely heart rate?

● 30/min
● 50/min
● 65/min
● 75/min
● 85/min

Answer: 50/min. The normal rhythm of the A-V node is 40 to 60 beats/min. Purkinje fibers have
a rhythm of 15 to 40 beats/min.

Sympathetic stimulation of the heart does which of the following?


● Releases acetylcholine at the sympathetic endings
● Decreases sinus nodal discharge rate
● Decreases excitability of the heart
● Releases norepinephrine at the sympathetic endings
● Decreases cardiac contractility

Answer: Releases norepinephrine at the sympathetic endings. Increased sympathetic stimulation


of the heart increases heart rate, atrial contractility, and ventricular contractility and increases
norepinephrine release at the ventricular sympathetic nerve endings. It does not release
acetylcholine. It does cause an increased sodium permeability of the A-V node, which increases
the rate of upward drift of the membrane potential to the threshold level for self-excitation, thus
increasing the heart rate.

Which statement best explains how sympathetic stimulation affects the heart?

● The permeability of the S-A node to sodium decreases


● The permeability of the A-V node to sodium decreases
● The permeability of the S-A node to potassium increases
● There is an increased rate of upward drift of the resting membrane potential of the S-A
node
● The permeability of the cardiac muscle to calcium decreases

Answer: There is an increased rate of upward drift of the resting membrane potential of the S-A
node. During sympathetic stimulation, the permeabilities of the S-A node and the A-V node
increase. In addition, the permeability of cardiac muscle to calcium increases, resulting in an
increased contractile strength. Furthermore, an upward drift of the resting membrane potential of
the S-A node occurs. Increased permeability of the S-A node to potassium does not occur during
sympathetic stimulation.

Sympathetic stimulation of the heart normally causes which condition?

● Acetylcholine release at the sympathetic endings


● Decreased heart rate
● Decreased rate of conduction of the cardiac impulse
● Decreased force of contraction of the atria
● Increased force of contraction of the ventricles

Answer: Increased force of contraction of the ventricles. Sympathetic stimulation of the heart
normally causes an increased heart rate, increased rate of conduction of the cardiac impulse, and
increased force of contraction in the atria and ventricles. However, it does not cause
acetylcholine release at the sympathetic endings because they contain norepinephrine.
Parasympathetic stimulation causes acetylcholine release. The sympathetic nervous system firing
increases in the permeability of the cardiac muscle fibers, the S-A node, and the A-V node to
sodium and calcium.

Which phase of the cardiac cycle follows immediately after the beginning of the QRS wave?

● Isovolumic relaxation
● Ventricular ejection
● Atrial systole
● Diastasis
● Isovolumic contraction

Answer: Isovolumic contraction. Immediately after the QRS wave, the ventricles begin to
contract, and the first phase that occurs is isovolumic contraction. Isovolumic contraction occurs
before the ejection phase and increases the ventricular pressure enough to mechanically open the
aortic and pulmonary valves.

Which of the following is correct about ECG?

● The mean vector of depolarization moves from negative to positive, from front to back,
from left to right
● The P wave represents atrial depolarization and repolarization
● The Q-T interval approximates the time of ventricular contraction
● The P-R interval includes ventricular repolarization
● The T wave always opposes QRS polarity

Answer: The Q-T interval approximates the time of ventricular contraction. The QRS complex
represents ventricular depolarization, and the T wave represents ventricular repolarization. The
Q-T interval is the time that the ventricle takes to depolarize-repolarize and represents the
ventricular contraction, as observed in the Wiggers’ diagram.

When recording lead aVL on an ECG, which is the positive electrode?

● Left arm
● Left leg
● Right leg
● Left arm + left leg
● Right arm + left leg

Answer: Left arm. By convention, the left arm is the positive electrode for lead aVL of an ECG.

When recording lead II on an ECG, the right arm is the negative electrode and the positive
electrode is the

● Left arm
● Left leg
● Right leg
● Left arm + left leg
● Right arm + left leg

Answer: Left leg. By convention, the left leg is the positive electrode for lead II of an ECG.

What is the normal QT interval?

● 0.03 second
● 0.13 second
● 0.16 second
● 0.20 second
● 0.35 second

Answer: 0.35 second. The contraction of the ventricles lasts almost from the beginning of the Q
wave and continues to the end of the T wave. This interval is called the Q-T interval and
ordinarily lasts about 0.35 second.

When recording lead II on an ECG, the negative electrode is the

● Right arm
● Left leg
● Right leg
● Left arm + left leg
● Right arm + left leg

Answer: Right arm. By convention, the right arm is the negative electrode for lead II of an ECG.

When recording aVF on an ECG, the negative electrode/s is/are in which area?

● Left arm, left leg


● Right arm, left arm
● Left leg
● Left leg, right leg
● Chest

Answer: Right arm, left arm. The augmented unipolar leads are obtained by connecting two
terminals to negative and one to positive. In case of aVF, the positive terminal is connected to the
left leg and the negative ones to the right and left arms.

A ventricular depolarization wave, when traveling 60 degrees in the frontal plane, will cause a
large positive deflection in which of the following leads?

● aVR
● aVL
● Lead I
● Lead II
● aVF

Answer: Lead II. Lead II has a positive vector at the 60-degree angle. The negative end of lead II
is at −120 degrees.

OLM: Physiology - Coronary Blood Flow

Which vasoactive agent is usually the most important controller of coronary blood flow?

● Adenosine
● Bradykinin
● Prostaglandins
● Carbon dioxide
● Potassium ions

Answer: Adenosine. Although bradykinin, prostaglandins, carbon dioxide, and potassium ions
serve as vasodilators for the coronary artery system, the major controller of coronary blood flow
is adenosine. Adenosine is formed as adenosine triphosphate degrades to adenosine
monophosphate. Small portions of the adenosine monophosphate are then further degraded to
release adenosine into the tissue fluids of the heart muscle, and this adenosine vasodilates the
coronary arteries.

The most likely cause of cardiac pain in acute ischemic coronary disease is an increase in the
extracellular concentration of the following:

● Adenosine
● Potassium
● Nitric oxide
● ATP
● Lactic acid

Answer: Lactic acid. Ischemia causes the cardiac muscle to release lactic acid, which stimulates
pain nerve endings in cardiac muscle, sending impulses through sensory afferent nerve fibers
into the central nervous system.

A 70-year-old man with a weight of 100 kg (220 lb) and a blood pressure of 160/90 mm Hg has
been told by his doctor that he has angina caused by myocardial ischemia. Which treatment
would be beneficial to this man?

● Increased dietary calcium


● Isometric exercise
● A beta-l receptor stimulator
● Angiotensin II infusion
● Nitroglycerin

Answer: Nitroglycerin. Several drugs have proven to be helpful to patients with myocardial
ischemia. Beta receptor blockers (not stimulators) inhibit the sympathetic effects on the heart and
are very helpful. ACE inhibition prevents the production of angiotensin II and thus decreases the
afterload effect on the heart. Nitroglycerin causes nitric oxide release, resulting in coronary
vasodilation. Isometric exercise increases blood pressure markedly and can be harmful and
increased dietary calcium would be of little benefit.

A 60-year-old man sustained an ischemia-induced myocardial infarction and died from


ventricular fibrillation. In this patient, what factor was most likely to increase the tendency of the
heart to fibrillate after the infarction?

● Low potassium concentration in the heart extracellular fluid


● A decrease in ventricular diameter
● Increased sympathetic stimulation of the heart
● Low adenosine concentration
● Decreased parasympathetic stimulation of the heart

Answer: Increased sympathetic stimulation of the heart. Increased sympathetic stimulation


excites the cardiac myocytes and makes them much more susceptible to fibrillation. High (not
low) potassium increases fibrillation tendency. An increase (not a decrease) in ventricular
diameter allows the cardiac muscle to be out of the refractory period when the cardiac impulse
next arrives and can increase the tendency to fibrillate. A low adenosine level probably only
causes some coronary constriction. Decreased parasympathetics allow the heart rate to increase
and has little to do with fibrillation.

What is one of the major causes of death after myocardial infarction?

● Increased cardiac output


● A decrease in pulmonary interstitial volume
● Fibrillation of the heart
● Increased cardiac contractility

Answer: Fibrillation of the heart. The major causes of death after myocardial infarction include a
decrease in cardiac output that prevents tissues of the body from receiving adequate nutrition and
oxygen delivery and prevents removal of waste materials. Other causes of death are pulmonary
edema, which reduces the oxygenation of the blood, fibrillation of the heart, and rupture of the
heart. Cardiac contractility decreases after a myocardial infarction.
Pharmacology - Angina Pectoris
A 46-year-old man was admitted to the hospital with a complaint of chest pain when he went to
the gym. Following an exercise stress test, he was prescribed a long-acting formulation of
metoprolol. Which of the following actions most likely mechanism by which atenolol prevents
angina?
● Increased myocardial oxygen supply
● Decreased atrioventricular conduction
● Decreased myocardial oxygen demand
● Increased systemic vascular resistance
● Increased left ventricular end-diastolic pressure
Correct answer: Decreased myocardial oxygen demand
A 63-year-old man recently diagnosed with exertional angina started treatment with verapamil,
one tablet daily. Which of the following cardiac and smooth muscle Ca2+ channels is most likely
the main site of action of this drug?
● Ligand-gated channels in cell membranes
● Store-operated channels in mitochondria
● Voltage-gated channels in the sarcoplasmic reticulum
● Voltage-gated channels in cell membranes
● Ligand-gated channels in the sarcoplasmic reticulum
Correct answer: Voltage-gated channels in cell membranes
A 55-year-old man complained of dizziness and palpitations shortly after taking a tablet of his
prescribed medication. The man was recently diagnosed with variant angina for which he had
started an appropriate therapy 4 days earlier. Which of the following actions most likely caused
the patient’s symptoms?
● Coronary vasodilation
● Decreased total peripheral resistance
● Increased venous return to the heart
● Decreased myocardial contractility
● Coronary steal phenomenon
Correct answer: Decreased total peripheral resistance
A 51-year-old plumber was brought to the emergency department because of severe chest pain
that had been ongoing for over 2 hours. The man had been suffering from chronic stable
exertional angina for 1 year and from duodenal ulcer for 3 months. His current medication
included isosorbide mononitrate and verapamil for angina and ibuprofen for a sprained ankle.
One week earlier, the patient stopped the antianginal medications because he had not had anginal
attacks during the past month. Which of the following events most likely triggered the patient’s
present chest pain?
● Chronic progression of ischemia uncovered by discontinuing therapy
● Famotidine-induced inhibition of verapamil metabolism
● Abrupt withdrawal from nitrate therapy
● Famotidine-induced inhibition of isosorbide mononitrate metabolism
● Reflex tachycardia due to nitrate therapy
Correct answer: Abrupt withdrawal from nitrate therapy
A 75-year-old man was admitted to the hospital because of feeling unwell. He had a long history
of asthma and severe systolic cardiac failure. Shortly after admission, the patient started
vomiting, then became agitated, verbally abusive, and disoriented in space and time. He was
telling the nurse that he heard loud voices cursing him. An electrocardiogram showed atrial
tachycardia with atrioventricular block. Which of the following drugs most likely caused the
patient’s symptoms?
● Captopril
● Digoxin
● Metoprolol
● Ethacrynic acid
● Albuterol
Correct answer: Digoxin
A 45-year-old African American woman with known hypertension has been receiving
propranolol and hydrochlorothiazide for several months. Now she has been diagnosed with
variant angina, and her physician wants to revise her therapy. Which of the following would be
the most appropriate therapeutic change to make at this time?
● Substitute nifedipine for the propranolol
● Add nitroglycerin to the therapeutic regimen
● Substitute captopril for the propranolol
● Substitute nifedipine for the hydrochlorothiazide
● Add captopril to the therapeutic regimen
Correct answer: Substitute nifedipine for the propranolol
Physiology - ECG - Arrhythmias
During a semiannual exam, the electrocardiogram (ECG) recording of a trained athlete reveals a
cardiac arrhythmia shown in the following trace. The abnormality seen in this lead II trace is due
to a change in which of the following electrical event(s) in the heart?

● Atrial depolarization
● Atrial repolarization
● Atrioventricular (AV) conduction time
● Ventricular depolarization
● Ventricular repolarization
Answer: Atrioventricular (AV) conduction time. This ECG trace shows a first-degree heart
block, which is indicated by long PR intervals without any missing QRS complexes. The PR
interval includes the recording of the depolarizations of the conduction tissue from the sinoatrial
(SA) node to the Purkinje fibers and indicates how long it takes the conduction cells (not
myocytes) to depolarize. The typical duration is 0.12-0.20 seconds, and trained athletes may
have a long PR interval due to increased vagal tone to the heart. Atrial depolarization is indicated
by the P wave of the ECG trace, which has a normal duration on this trace. Atrial repolarization
is not indicated by a visible wave of the ECG trace, as it occurs during the QRS complex.
Ventricular depolarization is indicated by the QRS complex and ST segment of the ECG trace,
which is normal on this trace. Ventricular repolarization is indicated by the T wave of the ECG
trace, which is normal.
A 58-year-old woman complains of shortness of breath on exertion. An electrocardiogram (ECG)
is ordered and her lead I trace is shown. Which of the following waves, segments, or intervals on
her ECG trace is outside of the normal range?

● P wave
● QRS complex
● ST B. P-R interval segment
● Q-T interval
Answer: P-R interval. The P-R interval is measured from the beginning of the P wave to the
beginning of the QRS complex and should be between 0.12 and 0.20 seconds in duration, which
is 3–5 small boxes on the ECG paper (each small box is 0.04 seconds). This patient’s P-R
interval is 0.36 seconds (9 small boxes), which is abnormally long, indicating a first-degree
atrioventricular block. The normal duration of the P wave is shorter than 0.12 seconds (3 small
boxes) and is normal for this patient. The normal duration of the QRS complex is less than 0.10
seconds (2.5 small boxes) and is normal for this patient. The normal duration of the S-T segment
is 0.8 to 0.12 seconds (2–3 small boxes) and is normal for this patient. The normal duration of
the Q-T interval is less than half of one R-R interval. So first calculate the duration between two
R waves, which is 0.92 seconds (23 small boxes), then measure the Q-T interval, which is 0.44
seconds (11 small boxes). Since the Q-T interval is less than half of the R-R interval, it is normal
for this patient.
A 13-year-old girl presents with recurrent episodes of syncope, which are induced by bending
over (e.g., to tie her shoes). Her electrocardiogram (ECG) reveals a short P-R interval and a delta
wave. Based on her presentation, her symptoms are most likely caused by abnormal accessory
conduction pathways through which structure of the heart?
● Atrial septum
● Atrioventricular node
● Atrioventricular ring
● Heart valves
● Ventricular septum
Answer: Atrioventricular ring. The syncope and ECG findings are characteristic of
Wolff-Parkinson-White (WPW) syndrome. In this syndrome, the heart has abnormal conduction
pathways between the atria and ventricles through the atrioventricular (AV) ring, which normally
does not conduct an electrical current; these pathways disrupt the normal conduction pathway
and electrical activity of the heart. The atrial and ventricular septa have normal conduction
pathways and are not disrupted by WPW. The AV node is the normal conduction pathway from
the atria to the ventricles. The heart valves do not conduct electrical currents.
Pharmacology - Acute Coronary Syndrome and Myocardial Infarction
On arrival at the hospital, the patient above appears pale and anxious and a little confused. The
attending physician diagnoses hypovolemic shock based on his presentation and
symptomatology. Which of the following would be the MOST APPROPRIATE next step in the
management of the patient at this stage?
● Take the patient to the operating theater for immediate surgery
● Administer analpha-1 adrenergic agonist to increase blood pressure and increase tissue
perfusion
● Administer matched blood to replace that lost through bleeding
● Administer a beta-1 agonist to increase heart rate and cardiac output and increase tissue
perfusion
● Place an oxygen mask on the patient and monitor his oxygen saturation
Correct answer: Administer matched blood to replace that lost through bleeding
Which of the following statements is correct with regard to the mechanism of action of nitrates in
angina?
● Nitrates bind directly to blood vessel receptors to cause vasodilation
● Nitrates open potassium channels on blood vessels to cause vasodilation
● Reflex bradycardia due to these agents decreases myocardial oxygen demand
● These drugs activate guanylyl cyclase to increase cGMP
● They increase cyclic AMP to cause vasodilation
Correct answer: These drugs activate guanylyl cyclase to increase cGMP
Which of the following drugs produces efficacious beta-adrenergic receptor activation with
minimal or no effects on alpha-adrenergic receptors?
● Dopamine
● Dobutamine
● Esmolol
● Isoproterenol
● Prazosin
Correct answer: Dobutamine
When administered to a healthy adult male, which of the following drugs is most likely to
produce a reflex-mediated tachycardia?
● Nesiritide
● Atropine
● High dose epinephrine
● Dobutamine
● Metoprolol
Correct answer: Nesiritide
Low-dose intravenous infusion of this agent is used to increase renal blood flow in management
of hypotensive states:
● Dobutamine
● Isoproterenol
● Dopamine
● Phenylephrine
● Norepinephrine
Correct answer: Dopamine
Which of the following drugs would be most likely to reduce preload by producing peripheral
venous pooling in a patient with heart failure and acute pulmonary edema?
● Digoxin
● Losartan
● Nitroglycerin Nesiritide
● Propranolol
● Prazosin
Correct answer: Nitroglycerin
Pharmacology - Antiarrhythmic Drugs
A 54-year-old woman complained to her physician of palpitations, insomnia, diarrhea, and
increased sweating for the past 3 weeks. Physical examination revealed a patient in moderate
distress with mild hand tremors and exophthalmos. Vital signs were blood pressure 146/62 mm
Hg, pulse 122 bpm, respirations 18/min. An EKG showed atrial tachycardia. Which of the
following drugs would be most appropriate to treat the patient’s arrhythmia?
● Quinidine
● Amiodarone
● Amlodipine
● Propranolol
● Sotalol
Correct answer: Propranolol
A 59-year-old woman presented with an abrupt onset of palpitations accompanied by a vague
complaint of “feeling ill”. Subsequent Holter monitoring revealed atrial fibrillation with a
ventricular response up to 152 bpm. The patient’s past history included primary
hyperparathyroidism and intermittent claudication for 3 years, apparently due to peripheral
occlusive arteriosclerosis. Which of the following drugs would be appropriate for the chronic
control of the patient’s arrhythmia?
● Digoxin
● Propranolol
● Verapamil
● Adenosine
● Phenytoin
Correct answer: Verapamil
A 65-year-old woman admitted to the emergency department with a myocardial infarction
developed sustained ventricular tachycardia. Neither amiodarone nor lidocaine was effective, and
the cardiologist decided to try another drug that acts mainly by blocking activated Na+ channels
and K+ channels. Which of the following drugs was most likely administered?
● Mexiletine
● Adenosine
● Sotalol
● Verapamil
● Procainamide
Correct answer: Procainamide
A 78-year-old man was admitted to the hospital because of dyspnea, a nonproductive cough, and
fever. The man had been receiving an antiarrhythmic drug for 2 months to treat refractory
supraventricular tachycardia. A chest x-ray showed diffuse bilateral infiltrates. Bacterial, fungal,
and viral cultures were negative. Which of the following drugs most likely caused the patient’s
pulmonary disorder?
● Flecainide
● Mexiletine
● Amiodarone
● Sotalol
● Procainamide
Correct answer: Amiodarone
A 62-year-old woman who had been suffering from atrial flutter for 3 months was admitted to
the hospital for cardioversion. She received an intravenous infusion of a drug for 10 minutes, and
a few minutes later the heart reverted to a normal sinus rhythm. Which of the following drugs
was most likely administered?
● Adenosine
● Amiodarone
● Ibutilide
● Mexiletine
● Lidocaine
Correct answer: Ibutilide
Physiology - Blood Flow Heart Sounds - Murmurs
Which of the following is associated with the first heart sound?
● Inrushing of blood into the ventricles as a result of atrial contraction
● Closing of the A-V valves
● Closing of the pulmonary valve
● Opening of the A-V valves
● Inrushing of blood into the ventricles in the early to middle part of diastole
Answer: Closing of the A-V valves. The first heart sound by definition is always associated with
the closing of the A-V valves. The heart sounds are usually not associated with opening of any of
the valves but with the closing of the valves and the associated vibration of the blood and the
walls of the heart. One exception is an opening snap in some mitral valves.
Which mechanism is associated with the third heart sound?
● Inrushing of blood into the ventricles as a result of atrial contraction
● Closing of the A-V valves
● Closing of the pulmonary valve
● Opening of the A-V valves
● Inrushing of blood into the ventricles in the early to middle part of diastole
Answer: Inrushing of blood into the ventricles in the early to middle part of diastole. The third
heart sound is associated with inrushing of blood into the ventricles in the early to middle part of
diastole. The next heart sound, the fourth heart sound, is caused by inrushing of blood in the
ventricles caused by atrial contraction. The first heart sound is caused by the closing of the A-V
valves, and the second heart sound is caused by the closing of the pulmonary and aortic valves.
The fourth heart sound is associated with which mechanism?
● In-rushing of blood into the ventricles from atrial contraction
● Closing of the A-V valves
● Closing of the pulmonary valve
● Opening of the A-V valves
● In-rushing of blood into the ventricles in the early to middle part of diastole
Answer: In-rushing of blood into the ventricles from atrial contraction. The fourth heart sound
occurs at the end of diastole and is caused by inrushing of blood into the ventricles due to atrial
contraction. The first heart sound is caused by closing of the A-V valves. The closing of the
aortic and pulmonary valves at the end of systole causes the second heart sound. This initiates a
vibration throughout the ventricles, aorta, and pulmonary artery. The third heart sound is caused
by inrushing of blood into the ventricles in the early to middle part of diastole.
An 80-year-old man at a local hospital was diagnosed with a heart murmur. A chest radiograph
showed an enlarged heart but no edema fluid in the lungs. The mean QRS axis of his ECG was
170 degrees. His pulmonary wedge pressure was normal. What is the diagnosis?
● Mitral stenosis
● Aortic stenosis
● Pulmonary valve stenosis
● Tricuspid stenosis
● Mitral regurgitation
Answer: Pulmonary valve stenosis. The mean electrical axis of the QRS of this patient is shifted
rightward to 170 degrees, which indicates that the right side of the heart is involved. Both aortic
stenosis and mitral regurgitation cause a leftward shift of the QRS axis. Mitral stenosis does not
affect the left ventricle, but in severe enough circumstances, it could cause an increase in
pulmonary artery pressure, which would cause an increase in pulmonary capillary pressure at the
same time. Tricuspid stenosis does not affect the right ventricle. Therefore, pulmonary valve
stenosis is the only condition that fits this set of symptoms.
A 40-year-old woman has been diagnosed with a heart murmur. A “blowing" murmur of
relatively high pitch is heard maximally over the left ventricle. The chest radiograph shows an
enlarged heart. Arterial pressure in the aorta is 140/40 mm Hg. What is the diagnosis?
● Aortic valve stenosis
● Aortic valve regurgitation
● Pulmonary valve stenosis
● Mitral valve stenosis
● Tricuspid valve regurgitation
Answer: Aortic valve regurgitation. Blowing murmurs of relatively high pitch are usually
murmurs associated with valvular insufficiency. The key pieces of data to identify this murmur
are the systolic and diastolic pressures. Aortic valve regurgitation typically has a high pulse
pressure, which is the systolic – diastolic pressure and in this case is 100 mm Hg. Also notice
that the diastolic pressure decreases to very low values of 40 mm Hg as the blood leaks back into
the left ventricle.
In which disorder will left ventricular hypertrophy normally occur?
● Pulmonary valve regurgitation
● Tricuspid regurgitation
● Mitral stenosis
● Tricuspid stenosis
● Aortic stenosis
Answer: Aortic stenosis. Left ventricular hypertrophy occurs when the left ventricle either has to
produce high pressure or when it pumps extra volume with each stroke. During aortic
regurgitation, extra blood leaks back into the ventricle during the diastolic period. This extra
volume must be expelled during the next heartbeat. During mitral regurgitation, some blood gets
pumped out into the aorta, while at the same time blood leaks back into the left atrium.
Therefore, the left ventricle is pumping extra volume with each heartbeat. During aortic stenosis,
the left ventricle must contract very strongly, producing high wall tension to increase the aortic
pressure to the values high enough to expel blood into the aorta. During mitral stenosis, the
ventricle is normal because the atrium produces the extra pressure to get blood through the
stenotic mitral valve.
Which heart murmur is heard during systole?
● Aortic valve regurgitation
● Pulmonary valve regurgitation
● Tricuspid valve stenosis
● Mitral valve stenosis
● Patent ductus arteriosus
Answer: Patent ductus arteriosus. Several diastolic murmurs can be heard easily with a
stethoscope. During diastole, aortic and pulmonary valve regurgitation occur through the
insufficient valves causing the heart murmur at this time. Tricuspid and mitral stenosis are
diastolic murmurs because blood flows through the restricted valves during the diastolic period.
Patent ductus arteriosus is heard in both systole and diastole.
An increase in left atrial pressure is most likely to occur in which heart murmur?
● Tricuspid stenosis
● Pulmonary valve regurgitation
● Aortic stenosis
● Tricuspid regurgitation
● Pulmonary valve stenosis
Answer: Aortic stenosis. Aortic stenosis has a very high ventricular systolic pressure. Diastolic
filling of the ventricle requires a much higher left atrial pressure. However, tricuspid stenosis and
regurgitation, pulmonary valve regurgitation, and pulmonary stenosis are associated with an
increase in right atrial pressure and should not affect pressure in the left atrium.
A 50-year-old woman at a local hospital has been diagnosed with a heart murmur. A murmur of
relatively low pitch is heard maximally over the second intercostal space to the right of the
sternum. The chest radiograph shows an enlarged heart. The mean QRS axis of the ECG is -45
degrees. What is the diagnosis?
● Mitral valve stenosis
● Aortic valve stenosis
● Pulmonary valve stenosis
● Tricuspid valve stenosis
● Tricuspid valve regurgitation
Answer: Aortic valve stenosis. This patient has a QRS axis of −45 degrees, indicating a leftward
axis shift. In other words, the left side of the heart is enlarged. In aortic valve stenosis, the left
side of the heart is enlarged because of the extra tension the left ventricular walls must exert to
expel blood out the aorta. Therefore, these symptoms fit with a patient with aortic stenosis. In
pulmonary valve stenosis, the right side of the heart hypertrophies, and in mitral valve stenosis,
there is no left ventricular hypertrophy. In tricuspid valve regurgitation, the right side of the heart
enlarges, and in tricuspid valve stenosis, no ventricular hypertrophy occurs.
A 40-year-old woman has been diagnosed with a heart murmur of relatively high pitch heard
maximally in the second intercostal space to the left of the sternum. The mean QRS axis of his
ECG is 150 degrees and the chest radiographs show an enlarged heart. The arterial blood oxygen
content is normal. What is the likely diagnosis?
● Aortic stenosis
● Aortic regurgitation
● Pulmonary valve regurgitation
● Mitral stenosis
● Tricuspid stenosis
Answer: Pulmonary valve regurgitation. This patient has a heart murmur heard maximally in the
“pulmonary area of cardiac auscultation.” The high pitch indicates regurgitation. The rightward
axis shift indicates that the right side of the heart has hypertrophied. The two choices that have a
rightward axis shift are pulmonary valve regurgitation and tetralogy of Fallot. In tetralogy of
Fallot, the arterial blood oxygen content is low, which is not the case with this patient. Therefore,
pulmonary valve regurgitation is the correct answer.
In which condition will right ventricular hypertrophy normally occur?
● Tetralogy of Fallot
● Mild aortic stenosis
● Mild aortic insufficiency
● Mitral stenosis
● Tricuspid stenosis
Answer: Tetralogy of Fallot. Right ventricular hypertrophy occurs when the right heart has to
pump a higher volume of blood or pump it against a higher pressure. Tetralogy of Fallot is
associated with right ventricular hypertrophy because of the increased pulmonary valvular
resistance, and this also occurs during pulmonary artery stenosis. Tricuspid insufficiency causes
an increased stroke volume by the right heart, which causes hypertrophy. However, tricuspid
stenosis does not affect the right ventricle.
Which heart murmur is only heard during diastole?
● Patent ductus arteriosus
● Aortic stenosis
● Tricuspid valve regurgitation
● Interventricular septal defect
● Mitral stenosis
Answer: Mitral stenosis. Mitral stenosis is heard during diastole only. Aortic stenosis, tricuspid
valve regurgitation, interventricular septal effect, and patent ductus arteriosus are clearly heard
during systole. However, patent ductus arteriosus is also heard during diastole.
A person with which condition is most likely to have low arterial oxygen content?
● Tetralogy of Fallot
● Pulmonary artery stenosis
● Tricuspid insufficiency
● Patent ductus arteriosus
● Tricuspid stenosis
Answer: Tetralogy of Fallot. In tetralogy of Fallot, there is an interventricular septal defect as
well as stenosis of either the pulmonary artery or the pulmonary valve. Therefore, it is very
difficult for blood to pass into the pulmonary artery and into the lungs to be oxygenated. Instead,
the blood partially shunts to the left side of the heart, thus bypassing the lungs. This situation
results in low arterial oxygen content.
A 2-year-old girl had an echocardiogram. The results indicated a thickened right ventricle. Other
data indicated that the patient had severely decreased arterial oxygen content and equal systolic
pressures in both cardiac ventricles. What condition is present?
● Interventricular septal defect
● Tetralogy of Fallot
● Pulmonary valve stenosis
● Pulmonary valve regurgitation
● Patent ductus arteriosus
Answer: Tetralogy of Fallot. In tetralogy of Fallot, an interventricular septal defect and increased
resistance in the pulmonary valve or pulmonary artery cause partial blood shunting toward the
left side of the heart without going through the lungs. This situation results in a severely
decreased arterial oxygen content. The interventricular septal defect causes equal systolic
pressures in both cardiac ventricles, which causes right ventricular hypertrophy and a wall
thickness very similar to that of the left ventricle.
Which heart murmur is only heard during diastole?
● Patent ductus arteriosus
● Mitral regurgitation
● Tricuspid valve stenosis
● Interventricular septal defect
● Aortic stenosis
Answer: Tricuspid valve stenosis. Mitral regurgitation and aortic stenosis are murmurs heard
during the systolic period. A ventricular septal defect murmur is normally heard only during the
systolic phase. Tricuspid valve stenosis and patent ductus arteriosus murmurs are heard during
diastole. However, a patent ductus arteriosus murmur is also heard during systole.
A 35-year-old woman visits her family practitioner for an examination. She has a blood pressure
of 160/75 mm Hg and a heart rate of 74 beats/min. Further tests by a cardiologist reveal that the
patient has moderate aortic regurgitation. Which set of changes would be expected in this
patient?
Pulse Pressure Systolic Pressure Stroke Volume
A ↑ ↑ ↑

B ↑ ↓ ↑

C ↑ ↓ ↓

D ↑ ↑ ↓

E ↓ ↓ ↓

F ↓ ↑ ↓

G ↓ ↑ ↑

H ↓ ↓ ↑

Answer: A. The difference between systolic pressure and diastolic pressure is the pulse pressure.
The two major factors that affect pulse pressure are the stroke volume output of the heart and the
compliance of the arterial tree. In patients with moderate aortic regurgitation (due to incomplete
closure of aortic valve), the blood that is pumped into the aorta immediately flows back into the
left ventricle. The backflow of blood into the left ventricle increases stroke volume and systolic
pressure. The rapid backflow of blood also results in a decrease in diastolic pressure. Thus,
patients with moderate aortic regurgitation have high systolic pressure, low diastolic pressure,
and high pulse pressure.
A 67-year-old man has an ejection fraction of 0.32, no cyanosis, a history of dilated
cardiomyopathy and heart failure, and a systolic murmur. What is your most likely diagnosis?
● Mitral stenosis
● Tetralogy of Fallot
● Mitral regurgitation
● Patent ductus arteriosus
● Tricuspid stenosis
Answer: Mitral regurgitation. Mitral regurgitation produces a systolic murmur. The history of
dilated cardiomyopathy suggests mitral regurgitation. The lack of cyanosis and age of the
patients eliminates tetralogy of Fallot as a likely diagnosis. The characteristics of the murmur
eliminates patent ductus arteriosus (machinery murmur), mitral and tricuspid stenosis (diastolic
murmurs).
The murmur in mitral stenosis is due to:
● Increased pulmonary pressures
● Narrowed outflow tract of the left ventricle
● Backflow from atria to the pulmonary vessels
● Narrowed mitral valve opening
● Increased pulmonary pressures and Narrowed mitral valve opening
Answer: Narrowed mitral valve opening. The diastolic murmur of mitral stenosis is due to the
narrow opening of the mitral valve, making it difficult for the blood to move from the left atria to
the left ventricle. A consequence of mitral stenosis is an increase in pressure of pulmonary
vessels, but this is not a cause of the murmur.
What is correct about interpretation of left ventricular pressure-volume loops in valve disease?
● Aortic stenosis shows a taller P-V loop with reduced preload
● Isovolumetric systolic period is lost in aortic regurgitation but preserved in mitral
regurgitation
● Aortic stenosis and regurgitation show a significantly increased afterload
● Mitral stenosis and regurgitation show a significantly increased afterload
● Isovolumetric diastolic period is lost in tricuspid stenosis
Answer: Aortic stenosis and regurgitation show a significantly increased afterload.
Abnormalities of the aortic valve (outflow tract of the left ventricle) associates with significant
increases in LV pressure and as such, increases in afterload (which does not occur in mitral
stenosis/regurgitation). Preload is unchanged in aortic stenosis. Isovolumetric systolic period is
lost in aortic and in mitral regurgitation. Tricuspid stenosis does not associate with loss of left
ventricular isovolumetric diastolic period.
A 6-month-old patient has a chest X-ray showing enlargement of the heart and blood work with
low PO2. Which situation best explains his condition?
● Patent ductus arteriosus
● A right-to-left shunt
● A left-to-right shunt
● Congenital tricuspid stenosis
● Interatrial septal defect
Answer: A right-to-left shunt. A right-to-left shunt as observed in tetralogy of Fallot results in
almost 2/3 of the blood passing from the right ventricle into aorta without oxygenation,
bypassing the lungs (right-to-left shunt). As a result, oxygen in the blood is low and the infant
develops cyanosis (blue baby) and rapidly shows a significant enlargement of the right ventricle.
Patent ductus arteriosus (a left-to-right shunt) will not show cyanosis at this stage and the blood
is usually hyper-oxygenated. An interatrial septal defect associates with a left-to-right shunt.
Tricuspid stenosis does not cause low oxygen in blood or associate with enlargement of the right
ventricle.
A 35-year-old woman visits her family practice physician for an examination. She has a mean
arterial blood pressure of 105 mm Hg and a heart rate of 74 beats/min. Further tests by a
cardiologist reveal that the patient has moderate aortic valve stenosis. Which set of changes
would be expected in this patient?
Pulse Pressure Stroke Volume Systolic Pressure

A ↑ ↑ ↑

B ↑ ↓ ↑

C ↑ ↓ ↓

D ↑ ↑ ↓

E ↓ ↓ ↓

F ↓ ↑ ↓

G ↓ ↑ ↑

H ↓ ↓ ↑

Answer: E. Pulse pressure is the difference between systolic pressure and diastolic pressure. The
two major factors that affect pulse pressure are the stroke volume output of the heart and the
compliance of the arterial tree. An increase in stroke volume increases systolic and pulse
pressure, whereas an increase in compliance of the arterial tree decreases pulse pressure.
Moderate aortic valve stenosis results in a decrease in stroke volume, which leads to a decrease
in systolic pressure and pulse pressure.
OLM: Pathology - Valvular Heart Diseases
Acute rheumatic fever is characterized by which type of hypersensitivity:
● Type 1
● Type 2
● Type 3
● Type 4
Correct answer: Type 2
What is the most common valve involved in acute rheumatic fever:
● Mitral valve
● Aortic valve
● Tricuspid valve
● Pulmonary valve
Correct answer: Mitral valve
What is the most common cause of death in acute rheumatic fever:
● Heart failure
● Myocarditis
● MI
● Pericarditis
● Cardiac tamponade
Correct answer: Heart failure
What is the most common cause of mitral regurgitation:
● Rupture of papillary muscle
● Infective endocarditis
● Left sided heart failure
● Right sided heart failure
● Mitral valve prolapse
Correct answer: Mitral valve prolapse
What is the most common cause of aortic regurgitation:
● Infective endocarditis
● Left sided heart failure
● Coarctation of aorta
● Marfan’s syndrome
● Acute Rheumatic fever
Correct answer: Infective endocarditis
What is Libman-Sacks endocarditis associated with:
● SLE
● CREST syndrome
● Infective endocarditis
● MI
● Kawasaki disease
Correct answer: SLE
What is the most common valve involved in infective endocarditis:
● Mitral valve
● Aortic valve
● Tricuspid valve
● Pulmonary valve
Correct answer: Mitral valve
Medical Microbiology - Infective Endocarditis/Myocarditis
A 55-year-old woman was admitted to the hospital with fever, malaise and loss of appetite of one
week duration. On examination, she was pale, her temperature was 101oF, heart rate
100 beats/min, and blood pressure 150/90 mm Hg. Several hemorrhages were noted on the
conjunctiva. A systolic murmur was heard at the base of the heart and an
echocardiograph demonstrated a tricuspid vegetation. A risk factor associated with this patient’s
infection is:
● intravenous drug user
● hospitalization
● dental work
● artificial heart valve
● her age
Correct answer: Intravenous drug user. Intravenous drug users are at risk for right-side IE.
Vegetations will present on the Tricuspid valve.
A 45-year-old missionary is working in a small rural village in Brazil. He presents to the hospital
with fever, fatigue, headaches, and a left eye swollen shut. microscopic examination of a blood
smear demonstrates intracellular amastigotes. The patient is diagnosed with:
● Chagas disease
● Toxoplasmosis
● Leishmaniasis
● Tuberculoid lepra
● Cutaneous anthrax
Correct answer: Chagas disease. Based on the lab demonstration of amastigotes, the history of
travel, and symptoms; the patient is infected with Trypanosoma cruzi: Chagas disease.
While on vacation, a 65 year old man develops fever, chills, night sweats, and malaise that began
insidiously and progressed over several days. He sought medical care at an urgent care
facility and was diagnosed with the flu. When the patient returns home, he has the same
symptoms plus weakness and dyspnea. He sees his primary physician who hears a new onset of
murmur on auscultation and admits the man to the hospital. Of note, the patient had extensive
dental work before his vacation. Three blood cultures grow an alpha hemolytic, catalase negative,
gram positive cocci. Serotyping of the isolate organism reveals no specific carbohydrates on the
cell wall. What is the most likely causative agent of this patient's disease?
● Staphylococcus aureus
● Streptococcus viridans
● Streptococcus pyogenes
● Staphylococcus epidermitidis
● Eikenella corrodens
Correct answer: Streptococcus viridans: the labs are confirming the infection with this bacterium.
A 60-year-old man was diagnosed clinically with subacute infective endocarditis. Routine blood
cultures failed to grow any organisms and the infection was initially described as culture
negative. However, after 7 days of culture, the causative agents were recovered. With which
group of organisms is he most likely infected?
● Coagulase positive Staphylococci
● HACEK group
● Coagulase negative Staphylococci
● Enterococci
● Streptococci viridans group
Correct answer: HACEK group: the cultures took more than 48hrs: known as negative cultures:
HACEK group
Pharmacology - Treatment of Infective Endocarditis
A 65-year-old man was admitted to the hospital for high fever, weakness and fatigue since last
four days. Physical examination noted hemorrhages under nails. Auscultation indicated a
murmur. Physician sent blood for culture and started treatment for infective endocarditis. Which
of the following antibiotics are recommended drugs to include in the regimen?
● Amoxicillin + clavulanic acid
● Tetracycline + rifampin
● Sulfonamides + trimethoprim
● Piperacillin + tazobactam + gentamicin
● Vancomycin + ampicillin + gentamicin
Correct answer: Vancomycin + ampicillin + gentamicin
A 65-year-old man was admitted to the hospital for high fever, weakness and fatigue since last
four days. Physical examination noted hemorrhages under nails. Auscultation indicated a
murmur. No history of any cardiac surgery and valve replacement. Blood culture report showed
Methicillin sensitive Staphylococcus aureus. Which of the following antibiotics are
recommended drugs to include in the regimen for native endocarditis?
● Amoxicillin + clavulanic acid
● Tetracycline + rifampin
● Sulfonamides + trimethoprim
● Nafcillin
● Vancomycin + ampicillin + gentamicin
Correct answer: Nafcillin
An 81-year-old woman with poor dentition was scheduled to have all of her remaining teeth
extracted for subsequent fitting of dentures. Past medical history was significant for numerous
infections of the oral cavity. The woman had been suffering from mitral stenosis with mild
cardiac insufficiency for 5 years. Which of the following drugs would be the most appropriate
agent to prescribe prophylactically to this woman before the extraction to prevent endocarditis?
● Amoxicillin
● Gentamicin
● Aztreonam
● Piperacillin
● Metronidazole
Correct answer: Amoxicillin
Which of the following mechanisms is most likely responsible for the resistance to vancomycin?
● D-ala to D-lac
● D-ala to L-ala
● D-Lac to L-ala
● Mutated NAG unit
● Mutated NAM unit
● Mutated transmembrane unit
Correct answer: D-ala to D-lac
Pathology - Diseases of Myocardium & Pericardium
What is the most common cause of myocarditis:
● Coxsackievirus B
● Acute RF
● SLE
● Trypanosoma cruzi
● Trichinella spiralis
Correct answer: Coxsackievirus B
What is the most common type of cardiomyopathy:
● Dilated
● Hypertrophic
● Restrictive
● Constrictive
Correct answer: Dilated
What is the most common cause of dilated cardiomyopathy:
● Idiopathic
● Thiamine deficiency
● Pregnancy
● Previous myocarditis
● Physiologic stress
Correct answer: Idiopathic
What is the most common type of Acute Pericarditis:
● Serous pericarditis
● Purulent pericarditis
● Fibrinous pericarditis
● Hemorrhagic pericarditis
● Caseous pericarditis
Correct answer: Fibrinous pericarditis
What is the most common malignant primary tumor in the heart:
● Myxoma
● Rhabdomyomas
● Rhabdomyosarcoma
● Mesothelioma
● Angiosarcoma
Correct answer: Angiosarcoma
Embryology - Development of the Heart and Vascular System
What is the direct embryological source of most of the heart?
● Ectoderm
● Neural crest cells only
● Lateral Plate mesoderm
● Endoderm, neural crest cells, visceral lateral plate mesoderm
● Epiblast and hypoblast cells
Correct answer: Lateral Plate mesoderm
Which forms most of the right subclavian artery?
● Aortic arch 1
● Aortic arch 2
● Aortic arch 3
● Right dorsal aorta
● Aortic arch 5
Correct answer: Right dorsal aorta
Pathology - Congenital Heart Diseases
What is the most common congenital heart disease:
● VSD
● ASD
● PDA
● Tetralogy of Fallot
● Coarctation of Aorta
Correct answer: VSD
What type of murmur does PDA have:
● Continuous harsh machinery murmur
● Early diastolic
● Pansystolic
● Mid to late diastolic
● Mid to late systolic
Correct answer: Continuous harsh machinery murmur
What is the most common cyanotic congenital heart disease:
● Tetralogy of Fallot
● Transposition of great arteries
● Truncus arteriosus
● Tricuspid atresia
● Total anomalous pulmonary venous return
Correct answer: Tetralogy of Fallot
Preductal coarctation of aorta is associated with:
● Turner syndrome
● Down syndrome
● Marfan syndrome
● Congenital bicuspid aortic stenosis
● Digeorge syndrome
Correct answer: Turner syndrome
Postductal coarctation of aorta is associated with:
● Turner syndrome
● Down syndrome
● Marfan syndrome
● Congenital bicuspid aortic stenosis
● Digeorge syndrome
Correct answer: Congenital bicuspid aortic stenosis

You might also like